You are on page 1of 81

Exam Title : TS7

Email : pandurangchormale@gmail.com

Contact : 9871159761

QUESTION 1.
Recently, the President re-promulgated an Ordinance to amend the Enemy Property Act. Consider the
following amendments with regard to Enemy Property Act.

1. It vest all rights, titles and interests over enemy property in the Custodian

2. It allows civil courts and other authorities to entertain disputes related to enemy property

Which of the statements given above is/are correct?

a) 1 only
b) 2 only
c) Both 1 and 2
d) Neither 1 nor 2
Correct Answer: A
Your Answer: Unanswered
Explanation

Ans. a

Explanation: The Ordinance bars civil courts and other authorities from entertaining cases against enemy
properties, or against actions of the central government or the Custodian under the Act.

Tikdams - Centre had designated some properties belonging to nationals of Pakistan and China as enemy
properties during the 1962, 1965 and 1971 conflicts. The name of the Act tells us that it relates to nationals
of other countries, so how could civil courts pass a verdict on it?

Source: Aug 29

http://www.civilsdaily.com/govt-looks-amend-enemy-property-act-whats/

http://www.thehindu.com/news/national/president-repromulgates-ordinance-to-amend-enemy-property-act/article904318

QUESTION 2.
With reference to the 'lion-tailed macaque', an Old World monkey found in India, which of the following
statements is/are correct ?

1. It is endemic to Western Ghats.

2. It is declared as endangered by IUCN red list.

Civilsdaily
Page 0
Email: hello@civilsdaily.com
Exam Title : TS7

Email : pandurangchormale@gmail.com

Contact : 9871159761

3. It is classified as protected species under Environment Protection Act 1986

Select the correct answer using the codes given below.

a) 1 and 2
b) 2 only
c) 1 and 3
d) 3 only
Correct Answer: A
Your Answer: Unanswered
Explanation

Ans. a

Explanation: All wildlife species are protected under Wildlife Protection Act 1972.

Tikdam: In such questions always try to see whether the Wildlife or Environment Protection Act applies.
Here the Wildlife Act will apply, in questions on pollution the Environment Protection Act will apply.
Eliminate all options with 3.

Titbits: The lion-tailed macaque is an Old World monkey endemic to the Western Ghats of South India. A
recent assessment for IUCN reports 3000-3500 of these animals live scattered over several areas in
Kerala. The lion-tailed macaque ranks among the rarest and most threatened primates.

Source: Aug 1

http://www.thehindu.com/news/cities/chennai/one-more-liontailed-macaque-at-zoo/article8926345.ece

http://www.civilsdaily.com/lion-tailed-macaque-vandalur-zoo-chennai/

QUESTION 3.
Recently, 'Green steel technology' was in the news. What is this technology all about?

a) It converts cast iron into environment-friendly green steel


b) It is simple modification in the conventional manufacturing process for steel
c) It is new technology that can be used to generate green energy
d) It will increase energy efficiency and sustainability of steel
Correct Answer: B
Your Answer: Unanswered
Explanation

Civilsdaily
Page 0
Email: hello@civilsdaily.com
Exam Title : TS7

Email : pandurangchormale@gmail.com

Contact : 9871159761

Ans. b

Explanation: Green Steel technology controls the injection of granulated waste tyres in conventional
Electric Arc Furnace (EAF) steelmaking and reduces the use of non-renewable coke

Benefits : It reduces the cost of EAF steelmaking and reduces the environmental footprint of the industry.

Tikdams Option c can be easily eliminated as it only talks about green energy and not steel. Option a can
also be eliminated as iron or steel by themselves are not harmful to environment, the process of making
them is. By this thinking option b is most likely as the answer.

Source: Aug 1

http://www.civilsdaily.com/turning-waste-tyre-green-steel/

http://www.thehindu.com/business/an-interview-veena-sahajwalla-director-centre-for-sustainable-materials-research-and

QUESTION 4.
Recently, IIT Kharagpur has taken up an initiative to facilitate the process of filing of potential Geographical
indications for "Goyna Bori". What is it?

a) A popular spice used extensively in the hilly cuisine


b) A wall painting art that uses interior coating colours
c) Famous traditional sweet of Arunachal pradesh
d) Fine art of using lentil paste mix to create exquisite designs
Correct Answer: D
Your Answer: Unanswered
Explanation

Ans. d

Explanation: Goyna Bori is the fine art of using lentil paste mix to create exquisite designs. It is distinctive
of certain parts of Bengal.

Titbits: With a distinct historical basis dating back several centuries, the art form has been elaborately
described by Rabindranath Tagore in his works. It is believed to have originated in Midnapore.

Source: Aug 1

http://www.civilsdaily.com/iit-kgp-helping-filing-gi-application-goyna-bori/

Civilsdaily
Page 0
Email: hello@civilsdaily.com
Exam Title : TS7

Email : pandurangchormale@gmail.com

Contact : 9871159761

http://www.business-standard.com/article/news-ians/iit-kharagpur-facilitating-filing-of-gi-application-for-goyna-bori-11607

http://www.civilsdaily.com/story/all-about-indias-gi-tags/

QUESTION 5.
India has decided to roll out the India Business Card for which of the following group of countries?

a) ASEAN
b) G20
c) SAARC
d) SCO
Correct Answer: C
Your Answer: Unanswered
Explanation

Ans. c

Explanation: The business card will be only given to prominent businessmen of the 8 SAARC countries
(including Pakistan).

Tikdams: SAARC and ASEAN are regional organisations which promote trade among their members,
hence answer will be one of the two.

Source: Aug 2

http://www.civilsdaily.com/india-readies-business-card-saarc-businessmen/

http://www.thehindu.com/news/national/india-readies-business-card-for-saarc-businessmen/article8929863.ece

QUESTION 6.
Recently, there was an outbreak of Japanese encephalitis following reports of the disease in Manipur.
Consider following statement w.r.t Japanese encephalitis.

1. It is flavivirus spread by mosquitoes and leads to inflammation of brain

2. It is related to dengue, yellow fever and the West Nile viruses

3. According to World Health Organisation, there is no cure for the disease

Select the correct answer using the codes given below.


a) 1 and 2 only
b) 2 only
c) 1 and 3 only
d) 1, 2 and 3 only

Civilsdaily
Page 0
Email: hello@civilsdaily.com
Exam Title : TS7

Email : pandurangchormale@gmail.com

Contact : 9871159761

Correct Answer: D
Your Answer: Unanswered
Explanation

Ans. d

Explanation: Japanese encephalitis virus (JEV) is a flavivirus related to dengue, yellow fever and West
Nile viruses, and is spread by mosquitoes. JEV is the main cause of viral encephalitis in many countries of
Asia with an estimated 68,000 clinical cases every year.

Titbits: Japanese encephalitis virus (JEV) is the most important cause of viral encephalitis in Asia. The first
case of Japanese encephalitis viral disease (JE) was documented in 1871 in Japan.

Source: Aug 2

http://www.civilsdaily.com/mizoramwakes-check-outbreak-japanese-encephalitis/

http://indianexpress.com/article/india/india-news-india/mizoram-takes-rapid-preventive-steps-to-check-outbreak-of-japan

QUESTION 7.
The 'nine-dash line' was sometimes in the news. Which of the following is related to it?

a) Indian Ocean
b) Sea of Japan
c) South China Sea
d) East China sea
Correct Answer: C
Your Answer: Unanswered
Explanation

Ans. c

Explanation: The nine-dash line stretches hundreds of kilometers south and east of the southerly Hainan
Island, covering the strategic Paracel and Spratly island chains. China buttresses its claims by citing 2,000
years of history when the two island chains were regarded as its integral parts.

Tikdam: If you knew the nine dash line was about conflict of China with neighbouring countries, then
options c and d become more likely. Some basic knowledge is required.

Civilsdaily
Page 0
Email: hello@civilsdaily.com
Exam Title : TS7

Email : pandurangchormale@gmail.com

Contact : 9871159761

Titbits: Recently, The Permanent Court of Arbitration (PCA) in The Hague, Netherlands, has ruled that
Chinas claims of historical rights over South China Sea (SCS) has no legal basis. South China Sea has
been in the news frequently.

Source:

http://www.civilsdaily.com/south-china-sea-dispute-can-you-please-tell-me-more/

http://indianexpress.com/article/world/world-news/chinas-nuclear-power-ambitions-sailing-into-troubled-waters-2945502

QUESTION 8.
India plans to develop the Agartala-Akhaura rail link, that will be a key railway connection from India to
which of the following country?

a) Myanmar
b) Bangladesh
c) China
d) Nepal
Correct Answer: B
Your Answer: Unanswered
Explanation

Ans. b

Explanation: Akhaura is in south eastern Bangladesh, an important railway junction which connects
Chittagong port, resource-rich Sylhet and Dhaka.

Civilsdaily
Page 0
Email: hello@civilsdaily.com
Exam Title : TS7

Email : pandurangchormale@gmail.com

Contact : 9871159761

Tikdam: Agartala is in Tripura, which is surrounded by Bangladesh on 3 sides. b seems most likely answer
in that case.

Titbits: The Agartala - Akhaura [Bangladesh] link will give full rail connectivity from east to west of
Bangladesh and will greatly facilitate India-Bangladesh trade and people to people ties.

Source: Aug 3

http://www.civilsdaily.com/new-delhi-agartala-rail-link-bangladesh-started/

http://www.livemint.com/Politics/9vjgWF7G21YHF60DKQdxPI/New-DelhiAgartala-rail-link-through-Bangladesh-can-bec

QUESTION 9.
Recently, 'methandienone' was in the news related to sports. What is it?

a) Energy drink enhances performance


b) a rubber material used in manufacturing of footballs
c) a banned anabolic steroid used in doping
d) a chemical substance that degrades performance
Correct Answer: C
Your Answer: Unanswered
Explanation

Ans. c

Explanation: Recently, wrestler Narsingh Yadav had tested positive for methandienone which is a banned
anabolic steroid.

Tikdams: When it relates to sports then doping is mostly relevant and has been in news.

Source: Aug 1

http://www.civilsdaily.com/tilted-nada-verdict-narsingh-yadavs-favour/

http://www.thehindu.com/sport/other-sports/what-tilted-nada-verdict-in-narsingh-yadavs-favour/article8928920.ece?utm_

QUESTION 10.
Consider the following statements regarding SAARC Terrorist Offences Monitoring Desk (STOMD).

Civilsdaily
Page 0
Email: hello@civilsdaily.com
Exam Title : TS7

Email : pandurangchormale@gmail.com

Contact : 9871159761

1. It was established in Colombo this year to collate, analyse and disseminate information on terror
strategies in SAARC region.

2.Pakistan is the only SAARC country which is not member of this desk.

Which of the statements given above is/are correct?

a) 1 only
b) 2 only
c) Both 1 and 2
d) Neither 1 nor 2
Correct Answer: D
Your Answer: Unanswered
Explanation

Ans. d

Explanation: The desk was established in Colombo in 1995 to collate, analyse and disseminate
information on terror offences, tactics, strategies and methods in the SAARC region.

All SAARC member countries are part of it.

Tikdams: Since it is a SAARC's initiative so consensus must be there from all countries.

Source: Aug 3

www.civilsdaily.com/rajnath-press-saarc-terror-desk/

http://www.thehindu.com/todays-paper/tp-national/rajnath-to-press-for-saarc-terror-desk/article8934876.ece

QUESTION 11.
Which of the following rightly defines the term 'Hydrological drought', which was sometimes in the news.

a) occurs when dry weather patterns dominates an area


b) occurs when deficiencies in surface and subsurface water supplies
c) occurs when hot and humid weather patterns in specific region
d) Occurs when the demand for water exceeds the supply
Correct Answer: B
Your Answer: Unanswered
Explanation

Ans. b

Civilsdaily
Page 0
Email: hello@civilsdaily.com
Exam Title : TS7

Email : pandurangchormale@gmail.com

Contact : 9871159761

Explanation: Hydrological drought is associated with deficiencies in surface and subsurface water
supplies. A meteorological drought often leads to hydrological drought. Generally it takes two successive
meteorological droughts before the hydrological drought sets in. There are two types of hydrological
droughts, a surface water drought and ground water drought.

Tikdams: Hydro means water. Drought means prolonged absence of something. So c and d can be easily
eliminated.

Source: Aug 3

http://www.civilsdaily.com/rains-good-dam-levels-continue-dip-karnataka/

http://www.thehindu.com/todays-paper/rains-are-good-but-dam-levels-continue-to-dip/article8935081.ece

QUESTION 12.
Consider the following statements with regard to Exclusive Economic Zone (EEZ).

1. A sea zone prescribed by the UN Convention on the Law of the Sea over which a state has special rights
regarding the exploration and use.

2.Its an area beyond and adjacent to the territorial sea, extending seaward to a distance of no more than
12 nautical miles

Which of the statements given above is/are incorrect?

a) 1 only
b) 2 only
c) Both 1 and 2
d) Neither 1 nor 2
Correct Answer: B
Your Answer: Unanswered
Explanation

Ans. b

Explanation: EEZ is an area beyond and adjacent to the territorial sea, extending seaward to a distance of
no more than 200 nautical miles.

Titbits: EEZ is a concept adopted at the Third United Nations Conference on the Law of the Sea (1982),
whereby a coastal State assumes jurisdiction over the exploration and exploitation of marine resources in
its adjacent section of the continental shelf, taken to be a band extending 200 nautical miles from the
shore.

Civilsdaily
Page 0
Email: hello@civilsdaily.com
Exam Title : TS7

Email : pandurangchormale@gmail.com

Contact : 9871159761

Source: Aug 4

www.civilsdaily.com/north-korea-fires-missile-japanese-waters/

http://www.thehindu.com/todays-paper/tp-international/north-korea-fires-missile-into-japanese-waters/article8939881.ec

QUESTION 13.
Recently, Goods and Services Tax Bill has been passed by the Parliament. What are the benefits of this
Bill?

1.It will make tax administration more complex because of uniformity in taxation

2.It will remove the cascading effect created by existing indirect taxes.

3.It will eliminate economic distortions in production, trade, and consumption

Select the correct answer using the codes given below.

a) 1 and 2
b) 2 only
c) 2 and 3
d) 1, 2 and 3 only
Correct Answer: C
Your Answer: Unanswered
Explanation

Ans. c

Explanation: GST is a single tax on the supply of goods and services, right from the manufacturer to the
consumer. Credits of input taxes paid at each stage will be available in the subsequent stage of value
addition, which makes GST essentially a tax only on value addition at each stage

Tikdam: Statement 1 about GST making tax system more complex is simply there to confuse people who
are not sure of GST principles. Reforms are intended to make the system simpler, not more complex.

Titbits: The GST is a tax reform that will simplify tax administration. In principle, it is the same as the
Value-added Tax (VAT) already adopted by all Indian States but with a wider base.

Source: Aug 4

www.civilsdaily.com/amended-gst-bill-gets-parliament-green-signal/

Civilsdaily
Page 0
Email: hello@civilsdaily.com
Exam Title : TS7

Email : pandurangchormale@gmail.com

Contact : 9871159761

http://www.thehindu.com/news/national/gst-bill-green-light-for-one-nation-one-tax/article8939374.ece?homepage=true

QUESTION 14.
Which of the following are statements associated with the cause of global warming.

1. Emission of greenhouse gases

2. Drastic increase in the emission of carbon dioxide

3. Extreme rainfall events

Select the correct answer using the codes given below.

a) 1 and 2
b) 2 only
c) 2 and 3
d) 1, 2 and 3 only
Correct Answer: A
Your Answer: Unanswered
Explanation

Ans. a

Explanation: Extreme rainfall events are caused by global warming, they dont cause global warming.

Source: Aug 4

www.civilsdaily.com/heavy-rainfall-not-linked-global-warming/

http://www.thehindu.com/todays-paper/tp-national/heavy-rainfall-not-linked-to-global-warming-says-iit-study/article89399

QUESTION 15.
'Bawariyas' tribes are sometimes in the news. They are natives of

a) Tamil Nadu
b) Andhra Pradesh
c) Madhya Pradesh
d) Rajasthan
Correct Answer: D
Your Answer: Unanswered

Civilsdaily
Page 0
Email: hello@civilsdaily.com
Exam Title : TS7

Email : pandurangchormale@gmail.com

Contact : 9871159761

Explanation

Ans. d

Explanation: Bawariyas are natives of Rajasthan and have now spread to UP, Haryana, Punjab and
Uttarakhand. The Census of India 1881 described them as a "hunting community" who derive their name
from the word bawar or noose with which they snare wild animals.

Source: Aug 3

http://www.civilsdaily.com/bulandshahr-gangrape-bawariya-tribe/

http://www.thehindu.com/news/national/other-states/bulandshahr-gangrape-case-turns-spotlight-on-nomadic-bawariya-t

QUESTION 16.
Consider following statements regarding Shubha Mudgal, who was in the news recently

1. She is a well-known Indian singer of Carnatic classical music

2. She got Rajiv Gandhi National Sadbhavana Award

3. She has received awards for her outstanding contribution towards the promotion of communal harmony,
peace and goodwill.

Select the correct answer using the codes given below.

a) 1 and 2
b) 2 only
c) 2 and 3
d) 1, 2 and 3 only
Correct Answer: C
Your Answer: Unanswered
Explanation

Ans. c

Explanation: Shubha Mudgal is a well-known Indian singer of Hindustani classical music, Thumri, Khayal,
Dadra. The Rajiv Gandhi National Sadbhavana Award is given on 20 August this year.

Titbits: UPSC asked person-in-news based questions in this years' prelims paper. Be prepared for any
new trend.

Civilsdaily
Page 0
Email: hello@civilsdaily.com
Exam Title : TS7

Email : pandurangchormale@gmail.com

Contact : 9871159761

Source: www.civilsdaily.com/shubha-mudgal-selected-rajiv-gandhi-national-sadbhavana-award/

http://indianexpress.com/article/lifestyle/art-and-culture/singer-shubha-mudgal-to-get-rajiv-gandhi-sadbhavana-award-29

QUESTION 17.
Recently, the ScatSat- 1 satellite, indigenously developed by ISRO, was in the news. What is the function
of this satellite?

a) Weather forecasting for tracking of cyclones


b) Multipurpose communication and broadcasting
c) To monitor India's borders for anti-infiltration operations
d) For oceanographic and meterological studies
Correct Answer: A
Your Answer: Unanswered
Explanation

Ans. a

Explanation: ScatSat-1 satellite is a weather forecasting satellite to monitor sea winds and help predict the
genesis of cyclones. The primary payload of ScatSat-1 is a scatterometer to keep a watch on the speed
and direction of ocean winds that indicate the formation and strengthening of cyclones. The satellite will
have the capacity to monitor temperature and humidity and transmit 48 images per day. It will replace
Oceansat-2.

Titbit: Currently India is dependent on NASAs ISS-RapidScat for prediction of cyclone forecasting and
weather prediction.

Source: Aug 5

www.civilsdaily.com/scatsat-1-help-predicting-cyclones/

http://www.thehindu.com/todays-paper/tp-national/scatsat-1-to-help-in-predicting-cyclones/article8945917.ece

QUESTION 18.
Recently, Doppler radar was in the news. What is the application of Doppler radar?

a) For air and terrestrial traffic control


b) For weather forecasting surveillance
c) For public health surveillance
d) For geological observations

Civilsdaily
Page 0
Email: hello@civilsdaily.com
Exam Title : TS7

Email : pandurangchormale@gmail.com

Contact : 9871159761

Correct Answer: B
Your Answer: Unanswered
Explanation

Ans. (b)

Explanation: Doppler radar is an observational tool for monitoring and predicting severe weather events
such as thunder storms, hailstorms, cyclones and tornados. It not only gives information about precipitation
but also about wind velocity.

Titbits: Doppler radar is designed to improve precision in long range weather forecasting and surveillance
using the Doppler effect. It works by bouncing a microwave signal off a target and analysing how the
objects motion has altered the frequency of the return signal.

Source: Aug 5

www.civilsdaily.com/doppler-radar-boost-weather-forecasting/

http://www.thehindu.com/todays-paper/tp-national/doppler-radar-to-boost-weather-forecasting/article8945918.ece

QUESTION 19.
Consider the following statements with regard to Ponzi scheme, which was sometimes in news.

1. It is a fraudulent investing scam promising high rates of return with little risk to investors.

2. Ponzi schemes fall under the regulatory purview of SEBI.

Which of the statements given above is/are correct?

a) 1 only
b) 2 only
c) Both 1 and 2
d) Neither 1 nor 2
Correct Answer: A
Your Answer: Unanswered
Explanation

Ans. (a)

Explanation: Ponzi scheme - A fraudulent investment operation where the operator pays returns to its
investors from new capital paid to the operators by new investors, rather than from profit earned through

Civilsdaily
Page 0
Email: hello@civilsdaily.com
Exam Title : TS7

Email : pandurangchormale@gmail.com

Contact : 9871159761

legitimate sources. It offers higher returns than other investments, in the form of short-term returns that are
either abnormally high or unusually consistent.

Titbits: Ponzi schemes do not fall under the regulatory purview of SEBI. The same is banned under Chit
Fund and Money Circulation (Banning) Act 1978, and the concerned state government is the enforcement
agency of the said law.

Source: Aug 9

www.civilsdaily.com/ponzi-schemes-not-purview-sebi/

http://www.thehindu.com/news/national/ponzi-schemes-not-under-our-purview-sebi/article8961345.ece

QUESTION 20.
Recently, Mihir Shah committee has recommended setting up a National Water Commission. Which of the
following organisations will get merged for this set up?

1. The Central Water Commission

2. The Central Ground Water board

3. The National Inland Water board

Select the correct answer using the codes given below.

a) 1 and 2
b) 2 only
c) 2 and 3
d) 1, 2 and 3 only
Correct Answer: A
Your Answer: Unanswered
Explanation

Ans. (a)

Explanation: Mihir Shah committee set up by the Ministry of Water Resources has recommended that a
National Water Commission(NWC) be set up.

NWC: Will be an autonomous body & will have a countrywide base and mandate, and greater
human-power

Civilsdaily
Page 0
Email: hello@civilsdaily.com
Exam Title : TS7

Email : pandurangchormale@gmail.com

Contact : 9871159761

Titbits: The Central Water Commission & the Central Ground Water Board have continued to function
un-reformed since their formation over several decades. National Water Commission will subsume these 2
organisations.

Source: Aug 10

www.civilsdaily.com/govt-panel-moots-national-water-commission/

http://www.business-standard.com/article/current-affairs/govt-panel-moots-national-water-commission-116081000040_1

QUESTION 21.
Science Cities have been set up by the Ministry of Culture, and are often in the news. Which of the
following cities is a Science City?

1. Central Science City, Delhi

2. Science City, Kolkata

3. Regional Science City, Lucknow

Select the correct answer using the codes given below.

a) 1 and 2
b) 2 only
c) 2 and 3
d) 1, 2 and 3 only
Correct Answer: C
Your Answer: Unanswered
Explanation

Ans. (c)

Explanation: Ministry of Culture has set up Science Cities across the country in a hub-and-spoke method
as part of efforts to have science and technology footprint in every State.

The Science Cities set up by the Ministry of Culture, Government of India are

1. Science City, Kolkata, West Bengal

2. Regional Science City, Lucknow, Uttar Pradesh

Civilsdaily
Page 0
Email: hello@civilsdaily.com
Exam Title : TS7

Email : pandurangchormale@gmail.com

Contact : 9871159761

3. Pushpa Gujral Science City, Kapurthala, Punjab

4. Gujarat Science City, Ahmadabad, Gujarat

Titbits: Each Science City would cater to scientific institutions/ labs located in different States in that
particular zone. The Science City would have the required infrastructure and other facilities, including a
convention centre, high-end research laboratories and hotels.

It would cater to scientific institutions of States covered by that zone and international researchers could
also make use of the available facilities.

Source: http://pib.nic.in/newsite/mbErel.aspx?relid=148082

QUESTION 22.
Consider the following statements with regard to 'Ecological footprint'.

1. It is a measure of the impact humans have on the environment

2. India ranks 3rd in its ecological footprint due to its high population levels despite a low per capita
consumption of natural resources

Which of the statements given above is/are Incorrect?

a) 1 only
b) 2 only
c) Both 1 and 2
d) Neither 1 nor 2
Correct Answer: D
Your Answer: Unanswered
Explanation

Ans. (d)

Explanation: Both statements are correct.

Ecological footprint is the measure of the impact humans have on the environment.

By WWF definition: A countrys ecological footprint is the sum of all the cropland, grazing land, forest and
fishing grounds required to produce the food, fibre and timber it consumes, to absorb the wastes emitted
when it uses energy and to provide space for infrastructure.

Titbits: India is ranked third in ecological footprint in the world, after China and USA.

Civilsdaily
Page 0
Email: hello@civilsdaily.com
Exam Title : TS7

Email : pandurangchormale@gmail.com

Contact : 9871159761

National Footprint Accounts 2016 by World Bank reported, Cropland and forest footprints were the largest
components of Indias overall ecological footprint, until the late 1980s. Carbon footprint took over in the late
2000s & Indias carbon footprint currently makes up 53% of the countrys overall Ecological Footprint.

Source: Aug 08

www.civilsdaily.com/world-already-finished-off-years-quota-natural-resources/

http://www.thehindu.com/sci-tech/energy-and-environment/earth-overshoot-day-worlds-finished-off-this-years-quota-of-n

QUESTION 23.
Consider the following statements with reference to 'Global Footprint Network', which was sometimes in the
news.

1. It is the principal body of IUCN and is based in the United States.

2. It develops and promotes tools for advancing sustainability, including the ecological footprint and
biocapacity.

3. The organization is headquartered in Oakland, California.

Select the correct answer using the codes given below.

a) 1 and 2
b) 2 only
c) 2 and 3
d) 1, 2 and 3 only
Correct Answer: C
Your Answer: Unanswered
Explanation

Ans. (c)

Explanation: Global Footprint Network, founded in 2003, is an independent think tank based in the United
States, Belgium and Switzerland. It is established as a charitable not-for-profit organization in each of those
three countries.

It develops and promotes tools for advancing sustainability, including the ecological footprint and
biocapacity, which measure the amount of resources we use and how much we have. These tools aim at
bringing ecological limits to the center of decision-making.

Civilsdaily
Page 0
Email: hello@civilsdaily.com
Exam Title : TS7

Email : pandurangchormale@gmail.com

Contact : 9871159761

Tikdams: If we know a little bit about IUCN then option 1 can easily be tackled, as there is no principal
body/organ working under IUCN.

Source:
http://www.thehindu.com/sci-tech/energy-and-environment/earth-overshoot-day-worlds-finished-off-this-years-quota-of-n

QUESTION 24.
India recently got its first tiger repository, where will it be located?

a) Periyar tiger reserve


b) Wildlife Institute of India
c) Forest Research Institute of India
d) Tadoba Tiger reserve
Correct Answer: B
Your Answer: Unanswered
Explanation

Ans. (b)

Explanation: The Wildlife Institute of India (WII) at Dehradun will house the countrys first repository on
tigers, under its new Tiger Cell.

Titbits: WII & National Tiger Conservation Authority (NTCA) did the work on tiger conservation and
population estimation, and in the process have generated a huge database.

Tracking: If a tiger skin is recovered at a place then a properly maintained database can be used to check
where the tiger might have come from. When a project needs environmental clearance, the spatial data can
be used to overlay the project plan on maps and check whether the project would interfere with wildlife
habitats that must not be disturbed

Source:

www.civilsdaily.com/country-gets-first-tiger-repository/

http://www.thehindu.com/sci-tech/energy-and-environment/india-gets-its-first-tiger-repository/article8953924.ece

QUESTION 25.
Consider the following statement regarding biofuels.

1. Biofuels are derived from biomass any matter derived from plants or animals.

Civilsdaily
Page 0
Email: hello@civilsdaily.com
Exam Title : TS7

Email : pandurangchormale@gmail.com

Contact : 9871159761

2. It has traditionally been used as fuel for energy production in the form of wood, charcoal or animal waste.

3. Green Diesel and Bio Diesel are second generation biofuels

Select the correct answer using the codes given below.

a) 1 and 2
b) 2 only
c) 2 and 3
d) 1, 2 and 3 only
Correct Answer: A
Your Answer: Unanswered
Explanation

Ans. (a)

Explanation: Green Diesel and Bio Diesel are first generation biofuels.

First generation biofuels are made from the sugars and vegetable oils found in arable crops, which can be
easily extracted using conventional technology.

Ethanol and biodiesel are the most widely used liquid biofuels. Ethanol can be produced through
fermentation and distillation from any raw material containing significant amounts of sugar (e.g sugar cane
or sugar beet) or starch (e.g maize, wheat or cassava).

Titbits: Biodiesel is produced by chemically combining vegetable oil or animal fat with an alcohol. It can be
produced for instance from rapeseed, soybean, palm, or coconut oils.

Source:

www.civilsdaily.com/2022-biofuel-will-become-50000-crore-business/

http://www.thehindubusinessline.com/economy/biofuel-biz-set-to-reach-rs-50k-cr-by-2022/article8968034.ece?utm_sour

QUESTION 26.
Cabinet Committee on Economic Affairs approved an R&D project for the development of Advanced Ultra
Super Critical (AUSC) technology, which will be useful for

a) for thermal power plants


b) for producing efficient renewable wind energy
c) for nuclear power plants
d) for advanced solar energy power plants

Civilsdaily
Page 0
Email: hello@civilsdaily.com
Exam Title : TS7

Email : pandurangchormale@gmail.com

Contact : 9871159761

Correct Answer: A
Your Answer: Unanswered
Explanation

Ans. (a)

Explanation: An R&D project for the development of Advanced Ultra Super Critical (AUSC) technology for
thermal power plants (TPPs).

AUSC technology for Thermal Power Plants will improve power plant efficiency, reduce carbon-dioxide
emissions and reduce coal consumption per unit of power generated.

Use of this technology in all future large power plants will ensure energy security for the country for a
longer period, along with a greener environment.

Titbits: The project will enable Indian industries to design, manufacture and commission higher efficiency
coal-fired power plants with indigenously developed technology and manufacturing processes. This will be
the first time large power plant equipment will be manufactured with advanced technologies, but without
any technological collaboration/licensing agreement with foreign companies.

Source: Aug 11

www.civilsdaily.com/grant-rd-project-ausc-technology/

http://www.business-standard.com/article/economy-policy/govt-approves-rs-1-554-crore-to-develop-technology-for-powe

QUESTION 27.
Recently, Maharashtra cabinet approved a project on climate-resilient agriculture. Consider the following
statements with regard to this project.

1. The project aims at suggesting changes in crop patterns and promoting extensions to agri businesses

2. The project will be supported by the World Bank

Which of the statements given above is/are correct?

a) 1 only
b) 2 only
c) Both 1 and 2
d) Neither 1 nor 2
Correct Answer: C

Civilsdaily
Page 0
Email: hello@civilsdaily.com
Exam Title : TS7

Email : pandurangchormale@gmail.com

Contact : 9871159761

Your Answer: Unanswered


Explanation

Ans. (c)

Titbits: Maharashtra cabinet allotted Rs. 1.5 crore from the Emergency Fund to create 23 special posts to
implement a project on climate-resilient agriculture.

- The project will be supported by the World Bank.

- Aim: Studying crop patterns as per climate change & promoting extensions to agri business, which will
help the State government tackle challenges faced by the farm sector.

In recent prelims, there have been states related questions (reports, schemes, policies etc.).

Source: Aug 11

www.civilsdaily.com/maharashtra-cabinet-approves-climate-resilient-agri-project/

http://www.thehindu.com/news/cities/mumbai/maharashtra-cabinet-approves-climateresilient-agri-project/article8972368

QUESTION 28.
The Union Cabinet approved amendments to the Maternity Benefit Act of 1961. Consider the following
statements with regard to amendments to the Act.

1. It increases paid leave for expectant mothers from 3 months to 6 and a half months.

2. Women adopting a newborn and those having babies through surrogacy will not be entitled for maternity
leave.

3. It facilitates "work from home" and "mandatory provision for crche" for factories with at least 50 workers.

Select the correct answer using the codes given below.

a) 1 and 2
b) 2 only
c) 1 and 3
d) 1,2 and 3 only
Correct Answer: C
Your Answer: Unanswered
Explanation

Civilsdaily
Page 0
Email: hello@civilsdaily.com
Exam Title : TS7

Email : pandurangchormale@gmail.com

Contact : 9871159761

Ans. (c)

Explanation: Union Cabinet approved amendments to the Maternity Benefit Act of 1961 to increase paid
leave for expectant mothers from 3 months to 6 and a half months.

The Act regulates the employment of women during the period of child birth, and provides maternity
benefits. The Act applies to factory, mines, plantations, shops and other establishments. In a first, women
adopting a newborn and those having babies through surrogacy will also be entitled for maternity leave for
three months.

Titbits: These amendments are applicable to factories with at least 10 workers & will help around 18 lakh
women workers in the organised sector.

In case of a woman who has two or more children, the maternity benefit will continue to be 12 weeks, which
cannot be availed before six weeks from the date of the expected delivery.

Source: Aug 11

www.civilsdaily.com/paid-maternity-leave-increased-6-months/

http://www.thehindu.com/news/national/paid-maternity-leave-increased-to-6-months/article8970558.ece

QUESTION 29.
Recently, BRICS Network University was in the news, which of the following statement correctly describes
it.

a) It will inter-connect academic courses across other Universities in BRICS countries


b) It is one of the best dedicated network systems University
c) It is devoted to the creation of masters and PhD programmes
d) It is specialised in connecting all Asian Universities with Indian Universities
Correct Answer: C
Your Answer: Unanswered
Explanation

Ans. (c)

Explanation: BRICS Network University (BRICS-NU) was first mooted in March 2015 and was scheduled
to be launched in 2016 but the member countries are yet to find common modalities for the unique venture

Technical issues: Curriculum, language differences among the students of member countries and nature of
infrastructure.

Civilsdaily
Page 0
Email: hello@civilsdaily.com
Exam Title : TS7

Email : pandurangchormale@gmail.com

Contact : 9871159761

Titbits: The network university is devoted to the creation of masters and PhD programmes

Six thematic groups set up in the network, matching 6 agreed priority areas

The thematic groups will gather and decide on the content of the masters and PhD programmes and they
are supposed to start academic and student exchanges in 2017

BRICS studies covers areas such as politics, international relations, cultures and languages

Source: Aug 12

http://www.civilsdaily.com/technical-issues-delay-launch-brics-university/

http://www.thehindu.com/news/national/technical-issues-delay-launch-of-brics-university/article8975336.ece

QUESTION 30.
Consider the following statements with regard to Pradhan Mantri Gram Sadak Yojana(PMGSY).

1. A Centrally- sponsored scheme to build rural road connectivity

2. It aims to provide roads in rural India with a population of 500 persons or above in plains and 250
persons or above in hill States

3. It is implemented by Ministry of Road Transport and Highways

Select the correct answer using the codes given below.

a) 1 and 2
b) 2 only
c) 1 and 3
d) 1, 2 and 3 only
Correct Answer: A
Your Answer: Unanswered
Explanation

Ans. (a)

Explanation: Pradhan Mantri Gram Sadak Yojana, a Centrally- sponsored scheme to build rural road
connectivity, launched in December 2000 under Ministry of Rural Development.

Civilsdaily
Page 0
Email: hello@civilsdaily.com
Exam Title : TS7

Email : pandurangchormale@gmail.com

Contact : 9871159761

It aims to provide all-weather roads to all unconnected habitations in rural India with a population of 500
persons or above (in plains) and 250 persons or above (in hill States).

Source: Aug 13

www.civilsdaily.com/cag-uncovers-roads-nowhere/

http://www.thehindu.com/news/national/cag-uncovers-roads-to-nowhere/article8982791.ece

QUESTION 31.
Recently, "Maglev technology" was in the news. Which of the following statement correctly describes it?

a) Transport method which runs on solar energy cells


b) Railway transport uses magnetic levitation to move vehicles without touching the ground
c) Air Transport technology for efficient use of fuel
d) Magnetic levitation technology used in satellites
Correct Answer: B
Your Answer: Unanswered
Explanation

Ans. (b)

Explanation: Maglev is a transport method that uses magnetic levitation to move vehicles without touching
the ground. A maglev train floats above the tracks on a cushion of magnetic field. A vehicle can travel along
a guideway using magnets to create both lift and propulsion, thereby reducing friction by a great extent and
allowing high speeds.

Source: Aug 15

http://www.business-standard.com/article/economy-policy/indian-railways-to-run-magnetic-trains-soon-116081500795_1

QUESTION 32.
'Global innovation index- (GII) ' is sometimes in the news. Which of the following are important parameters
for calculating GII?

1. Institutions

2. Human capital and research

3. Infrastructure

Civilsdaily
Page 0
Email: hello@civilsdaily.com
Exam Title : TS7

Email : pandurangchormale@gmail.com

Contact : 9871159761

4. Market sophistication

Select the correct answer using the codes given below.

a) 1 and 2
b) 2 only
c) 1, 2 and 3
d) 1, 2, 3 and 4 only
Correct Answer: D
Your Answer: Unanswered
Explanation

Ans. (d)

Explanation: GII is co-published by Cornell University, INSEAD, and the World Intellectual Property
Organization (WIPO, an agency of the United Nations). The core of the GII Report consists of a ranking of
world economies innovation capabilities and results. The GII helps to create an environment in which
innovation factors are continually evaluated.

Titbits: Five input parameters are (1) Institutions, (2) Human capital and research, (3) Infrastructure, (4)
Market sophistication, and (5) Business sophistication.

Source: Aug 16

www.civilsdaily.com/india-climbs-innovation-ranking/

http://www.livemint.com/Politics/lATexj4C2o8ynCSRyM1VeL/India-climbs-15-spots-in-innovation-ranking.html

QUESTION 33.
The Regional Comprehensive Economic Partnership (RCEP) is related with

a) G20
b) BRICS
c) European Union
d) ASEAN
Correct Answer: D
Your Answer: Unanswered
Explanation

Ans. (d)

Civilsdaily
Page 0
Email: hello@civilsdaily.com
Exam Title : TS7

Email : pandurangchormale@gmail.com

Contact : 9871159761

Explanation: Regional Comprehensive Economic Partnership (RCEP) is a free trade agreement (FTA)
between ASEAN and the 6 states with which ASEAN has existing FTAs (Australia, China, India, Japan,
South Korea and New Zealand).

RCEP negotiations were formally launched in November 2012 at the ASEAN Summit in Cambodia.

Titbits: It was frequently in the news, India is one of the existing FTA member seeking RCEP.

Source: Aug 16

www.civilsdaily.com/call-remove-ip-clauses-trade-pact/

http://www.thehindu.com/news/national/call-to-remove-ip-clauses-from-trade-pact/article8991508.ece

QUESTION 34.
Consider the following statement with regard to 'Goods and Services Tax Network (GSTN)'

1. It is a non-profit entity that is building the information technology backbone for the goods and services
tax.

2. Analytics based on data filed by millions of taxpayers, will help in plugging leakages.

Which of the statements given above is/are incorrect?

a) 1 only
b) 2 only
c) Both 1 and 2
d) Neither 1 nor 2
Correct Answer: D
Your Answer: Unanswered
Explanation

Ans. (d)

Explanation: GSTN is the information technology backbone that will implement the new GST regime. It will
become a data analytics powerhouse after roll out. Once sufficient amount of data is generated, it will be
able to generate analytics based on the requirements of various stakeholders.

Advantages: These analytics, based on data filed by millions of taxpayers, will help in plugging leakages. It
will identify economic trends and ensure more focused economic-policymaking.

Source: Aug 17

Civilsdaily
Page 0
Email: hello@civilsdaily.com
Exam Title : TS7

Email : pandurangchormale@gmail.com

Contact : 9871159761

http://www.civilsdaily.com/gst-platform-become-analytics-powerhouse/

http://www.business-standard.com/article/economy-policy/gst-platform-to-become-analytics-powerhouse-116081601354

QUESTION 35.
Consider following statements with reference to Quantum Experiments at Space Scale, (QUESS) satellite,
which was recently in the news.

1. NASA launched the worlds first quantum (QUESS) satellite.

2. It helps to establish hack-proof communications between space and the ground

3. It is impossible to wiretap, intercept or crack the information transmitted through QUESS satellite.

Select the correct answer using the codes given below.

a) 1 and 2
b) 2 only
c) 2 and 3
d) 1, 2 and 3 only
Correct Answer: C
Your Answer: Unanswered
Explanation

Ans. (c)

Explanation: China launched the worlds first quantum satellite, which will help it establish hack-proof
communications between space and the ground. QUESS is designed to establish hack-proof quantum
communications by transmitting un-crackable keys from space to the ground. It will enable secure
communications between Beijing and Urumqi.

Titbits: It has ultra-high security as a quantum photon can neither be separated nor duplicated. Hence it is
impossible to wiretap, intercept or crack the information transmitted through it. It holds enormous prospects
in the field of secure communications.

Source: Aug 16

http://www.civilsdaily.com/china-launches-hack-proof-communications-satellite/

http://www.business-standard.com/article/international/china-launches-world-s-1st-hack-proof-communications-satellite-

Civilsdaily
Page 0
Email: hello@civilsdaily.com
Exam Title : TS7

Email : pandurangchormale@gmail.com

Contact : 9871159761

QUESTION 36.
Which of the following countries are not member states of Nuclear Suppliers Group (NSG).

1. China

2. India

3. Pakistan

4. Turkey

5. North Korea

Select the correct answer using the codes given below.

a) 1 and 2
b) 2 and 3 only
c) 2, 3 and 5
d) 2 and 4 only
Correct Answer: C
Your Answer: Unanswered
Explanation

Ans. (c)

Explanation: Nuclear Suppliers Group (NSG) is a group of nuclear supplier countries that seek to prevent
nuclear proliferation by controlling the export of materials, equipment and technology that can be used to
manufacture nuclear weapons. The NSG was founded in response to the Indian nuclear test in May 1974.
China and Turkey are member states to NSG.

Source: Aug 17

http://www.civilsdaily.com/pakistan-mutual-ban-nuclear-tests/

http://www.thehindu.com/news/national/pakistan-for-mutual-ban-on-nuclear-tests/article8995496.ece

QUESTION 37.
Which of the following is/are nodal agency in charge of National Clean Ganga Mission?

a) Ministry of Urban Development


b) Ministry of Water Resources
c) Niti Ayog
d) Ganga Action Task Force

Civilsdaily
Page 0
Email: hello@civilsdaily.com
Exam Title : TS7

Email : pandurangchormale@gmail.com

Contact : 9871159761

Correct Answer: B
Your Answer: Unanswered
Explanation

Ans. (b)

Explanation: The Union Water Resources Ministry is the nodal agency in charge of implementing the Rs.
20,000-crore National Clean Ganga Mission by 2022.

Titbits: The Govt plans to draft a Bill to regulate a river the Ganga Act. It will be drawn on the line of the
National Highways Act and will allow the Centre final say over States during disputes over management of
its water.

Source: Aug 17

www.civilsdaily.com/draft-ganga-act-draw-highways-act/

http://www.thehindu.com/news/national/draft-ganga-act-to-draw-from-highways-act/article8995510.ece

QUESTION 38.
The Railway Ministry has announced four new categories of trains.

Consider the following pairs:

Trains Description

1. Antyodaya Express : Long-distance, fully unreserved

2. Deen Dayalu Coaches : Double-Decker Air-conditioned reserved overnight trains

3. Humsafar : A fully third AC train

4. UDAY : Long-distance trains for unreserved travel

Which of the pairs given above are correctly matched?

a) 1, 2 and 4 only
b) 2, 3 and 4 only
c) 2 and 4 only
d) 1 and 3 only
Correct Answer: D
Your Answer: Unanswered

Civilsdaily
Page 0
Email: hello@civilsdaily.com
Exam Title : TS7

Email : pandurangchormale@gmail.com

Contact : 9871159761

Explanation

Ans. (d)

Explanation: Antyodaya Express: A long-distance, fully unreserved, super-fast train service, to be


introduced for the common man, will operate on dense routes

Deen Dayalu Coaches: To be added to some long-distance trains for unreserved travel to enhance the
carrying capacity

Humsafar: A fully third AC train

UDAY: (Utkrisht Double-Decker Air-conditioned Yatri) will be overnight trains plying on the busiest routes to
increase capacity by 40%

Tikdams: Option 2 can guessed, as Deen Dayalu by name itself signifies that it will be an unreserved train
and particularly for poor people. So, by eliminating option 2 we get option 'd' as answer easily.

Source:

http://www.civilsdaily.com/suresh-prabhu-announces-four-new-trains/

http://www.thehindu.com/news/national/suresh-prabhu-new-trains-antyodaya-express-utkrisht-doubledecker-aircondition

QUESTION 39.
Recently, Finance Ministry accepted Railway Ministry's proposal to merge Railway Budget with the General
Budget, ending a 92 year old practice. Which of the following committee has suggested this idea of
merging railway budget?

a) Bibek Debroy committee


b) Lodha Committee
c) Subramanian Committee
d) Suresh Prabhu Commiittee
Correct Answer: A
Your Answer: Unanswered
Explanation

Ans. (a)

Explanation: Two-member committee comprising Niti Aayog member Bibek Debroy and Kishore Desai
recommended the Railway budget exercise be scrapped.

Civilsdaily
Page 0
Email: hello@civilsdaily.com
Exam Title : TS7

Email : pandurangchormale@gmail.com

Contact : 9871159761

Source: Aug 14

http://www.thehindu.com/news/national/railway-budget-to-be-merged-with-general-budget-from-2017/article8988441.ec

QUESTION 40.
Recently, Sakshi Malik bagged an Olympic medal. Consider the following statements with regard to this
historic victory.

1. She scripted history by becoming the first woman wrestler from India to bag an Olympic medal.

2. She is the only fourth female athlete from the country to earn a medal in Olympics

Which of the statements given above is/are incorrect?

a) 1 only
b) 2 only
c) Both 1 and 2
d) Neither 1 nor 2
Correct Answer: D
Your Answer: Unanswered
Explanation

Ans. (d)

Explanation: Both statements are correct. She is 1st women wrestler and 4th female athlete to earn medal
in Olympic.

Titbits: UPSC trends towards person-in-news based question, so with changing nature of questions.

Source: Aug 18

www.civilsdaily.com/result-12-years-hard-work-says-sakshi-malik/

http://www.thehindu.com/sport/other-sports/bronze-medal-is-a-result-of-my-12-years-of-hard-work-sakshi/article9000424

QUESTION 41.
Consider following statements with reference to Financial Stability and Development Council (FSDC).

1. Aim of the FSDC is to strengthen and institutionalize the mechanism for maintaining financial stability
and enhancing inter-regulatory coordination

Civilsdaily
Page 0
Email: hello@civilsdaily.com
Exam Title : TS7

Email : pandurangchormale@gmail.com

Contact : 9871159761

2. FSDC has replaced the High Level Coordination Committee on Financial Markets, which was facilitating
regulatory coordination

3. The Chairman of the FSDC is the RBI Governor.

Select the correct answer using the codes given below.

a) 1 and 2
b) 2 only
c) 2 and 3
d) 1, 2 and 3 only
Correct Answer: A
Your Answer: Unanswered
Explanation

Ans. (a)

Explanation: Recently, the Report of the Working Group on Development of Corporate Bond Market in
India has been submitted to RBI Governor Raghuram Rajan.

Aim of the FSDC is to strengthen and institutionalize the mechanism for maintaining financial stability and
enhancing inter-regulatory coordination.

The Chairman of the FSDC is the Finance Minister of India.

Source:
http://www.thehindu.com/business/markets/panel-suggests-corporate-bond-index-easier-norms-for-fpis/article9003894.e

QUESTION 42.
The World Intellectual Property Organizations (WIPO) latest Global Innovation index has come out with
India lagging in some parameters.

Which of the following are parameters in which India lags behind other nations?

1. Government effectiveness

2. Infrastructure

3. Knowledge creation

Which of the statements given above is/are correct?

Civilsdaily
Page 0
Email: hello@civilsdaily.com
Exam Title : TS7

Email : pandurangchormale@gmail.com

Contact : 9871159761

a) 1 and 2 only
b) 2 only
c) 2 and 3 only
d) 1, 2 and 3 only
Correct Answer: D
Your Answer: Unanswered
Explanation

Ans. (d)

Explanation: In the Global Innovation Index (GII) 2016, India continues to lag in several areas important
for growth of business and industry such as government effectiveness, infrastructure and knowledge
creation.

India is ranked at 66th position.

Titbits: The annual report, published by the World Intellectual Property Organization (WIPO), assesses
countries on seven major indicators with 84 sub-indicators that range across the political, social,
educational and financial spectrum.

Source: http://www.civilsdaily.com/india-climbs-innovation-ranking/

QUESTION 43.
Consider follwing statements with regard to the 'Internet Corporation for Assigned Names and Numbers'
(ICANN).

1. It is a Japan-based inter-governmental organisation

2. It was created in 1998 to take over the task of assigning web addresses

3. Recently, the U.S. handed over power of the Internets domain naming system (DNS) to ICANN

Which of the statements given above is/are correct?

a) 1 and 2 only
b) 2 only
c) 2 and 3 only
d) 1, 2 and 3 only
Correct Answer: C
Your Answer: Unanswered
Explanation

Civilsdaily
Page 0
Email: hello@civilsdaily.com
Exam Title : TS7

Email : pandurangchormale@gmail.com

Contact : 9871159761

Ans (c)

Explanation: Internet Corporation for Assigned Names and Numbers (ICANN) is a Los Angeles-based
non-profit organisation.

It was created in 1998 to take over the task of assigning web addresses (DNS - Domain naming system).

Source:
http://www.thehindu.com/sci-tech/technology/us-set-to-hand-over-internets-naming-system-to-icann/article9003177.ece

www.civilsdaily.com/lets-know-icann/

QUESTION 44.
What is Access to Justice project, which was sometimes in the news?

1. It's objective is to address the legal needs of the marginalised and vulnerable sections of society.

2. It is being implemented only in the eight North-Eastern States for five years

3. The Department of Justice has been implementing project for Marginalised People with the UNDPs
support

Which of the statements given above is/are correct?

a) 1 and 2 only
b) 2 only
c) 1 and 3 only
d) 1, 2 and 3 only
Correct Answer: C
Your Answer: Unanswered
Explanation

Ans. (c)

Explanation: Its objective is to address the legal needs of the marginalised and vulnerable sections of
society, particularly women, children, and SC/ST communities, who do not have the requisite means to
ensure that their rights are guaranteed.

It is being implemented in the eight North-Eastern States, and Jammu and Kashmir, at a total cost of Rs.30
crore for five years (2012-17).

Civilsdaily
Page 0
Email: hello@civilsdaily.com
Exam Title : TS7

Email : pandurangchormale@gmail.com

Contact : 9871159761

The Department of Justice has been implementing Access to Justice for Marginalised People with the
UNDPs support.

Source: www.civilsdaily.com/aims-objectives-access-justice-project/

QUESTION 45.
According to latest analysis air pollution report released by the Central Pollution Control Board (CPCB),
which of the following cities had highest number of good quality air days recorded?

a) Allahabad
b) Varanasi
c) Gwalior
d) Coimbatore
Correct Answer: D
Your Answer: Unanswered
Explanation

Ans. (d)

Explanation: According to latest analysis air pollution report released by the Central Pollution Control
Board (CPCB). Coimbatore and Rajkot had highest number of good quality days, while Varanasi, Gwalior
and Allahabad didnt have even one good air quality day among all the days when their air quality was
monitored.

Source: www.civilsdaily.com/41-indian-cities-bad-air-quality-cpcb-survey-finds/

QUESTION 46.
'State of ICT in Asia and the Pacific 2016: Uncovering the Widening Broadband' report is published by

a) Internet Corporation for Assigned Names and Numbers (ICANN)


b) BRICS
c) SAARC
d) UN Economic and Social Commission for Asia and the Pacific (ESCAP)
Correct Answer: D
Your Answer: Unanswered
Explanation

Ans. (d)

Explanation: State of ICT in Asia and the Pacific 2016: Uncovering the Widening Broadband Divide
report is published by the United Nations Economic and Social Commission for Asia and the Pacific

Civilsdaily
Page 0
Email: hello@civilsdaily.com
Exam Title : TS7

Email : pandurangchormale@gmail.com

Contact : 9871159761

(ESCAP).

Titbits: India ranks a low 39th in terms of fixed broadband adoption among Asia Pacific countries, with just
1.3% of its citizens subscribing to such a service in 2015.

India ranks lower than countries such as Bhutan, Sri Lanka and Bangladesh in fixed broadband
subscriptions per 100 inhabitants in ESCAP countries in 2015.

Tikdams: SAARC can be straightforwardly eliminated, as report talks about ICT in Asia and Pacific and
SAARC is a regional forum for promoting development specifically in South Asia. Use a similar logic for
BRICS. This leaves options a and d.

Source: www.civilsdaily.com/india-ranks-39th-asia-pacific-fixed-broadband/

QUESTION 47.
What is 'Project Alloy', which was frequently in the news.

a) Project developed by Artificial intelligence


b) A device that creates merged reality, a type of virtual reality
c) First super-computer by NASA runs on solar energy
d) Project to find rare super metals on Mars
Correct Answer: B
Your Answer: Unanswered
Explanation

Ans. (b)

Explanation: Project Alloy is device that creates merged reality.

It is a headset that uses the RealSense technology enabling people to use their hands to interact with
elements of the virtual world

Titbits: Merged reality is a new way of experiencing virtual reality and real world together, using
cutting-edge technology, which is more dynamic and natural, and allows people to do things that are now
impossible.

Source:

www.civilsdaily.com/intel-unveils-merged-reality-project-alloy-device/

http://www.thehindu.com/business/Industry/intel-unveils-merged-reality-with-project-alloy-device/article9008432.ece

Civilsdaily
Page 0
Email: hello@civilsdaily.com
Exam Title : TS7

Email : pandurangchormale@gmail.com

Contact : 9871159761

QUESTION 48.
Consider following statements about 'CFL technology'.

1. CFL lamp works through a gas discharge

2. Reduced size, cost of CFL lamps and improved lighting efficiency by 75 per cent makes it more efficient.

3. CFLs are four times more efficient and last up to 10 times longer than incandescents.

Which of the statements given above is/are correct?

a) 1 and 2 only
b) 2 only
c) 2 and 3 only
d) 1, 2 and 3 only
Correct Answer: D
Your Answer: Unanswered
Explanation

Ans. (d)

Explanation:

CFL lamp: Works through a gas discharge.

Disadvantage of Tungsten lamp - Energy is lost due to the heating required by the tungsten filament.

Advantages of CFL: Allow for electronics to fit into the small volume of the base below the gas tube.
Reduced size & cost of CFL lamps & improved lighting efficiency by 75%.

Use of CFLs instead of traditional lighting, in the last 25 years, has saved the world 73,000 Terawatt-hours
of energy and almost 5.7 trillion litres of gas, and has helped decrease carbon dioxide emissions by 49.5
billion metric tonnes.

Source: www.civilsdaily.com/lets-know-cfl-technology/

QUESTION 49.
Which of the following countries is the worlds biggest tech start-up hub, according to a recently published
study of Assocham.

a) China
b) USA
c) India
d) Japan

Civilsdaily
Page 0
Email: hello@civilsdaily.com
Exam Title : TS7

Email : pandurangchormale@gmail.com

Contact : 9871159761

Correct Answer: B
Your Answer: Unanswered
Explanation

Ans. (b)

Explanation: India is home to the third largest number of technology driven startups in the world, with the
US and the UK occupying the top two positions, according to a report. The study was done by Assocham in
association with Thought Arbitrage Research Institute.

Titbits: Top Indian cities are Bengaluru, Delhi NCR, Mumbai, Hyderabad and Chennai.

Synergising Startup India with Make in India and Digital India initiatives has the potential to expand the
domestic ecosystem for new entrepreneurs

Source: Aug 22

http://indianexpress.com/article/technology/tech-news-technology/india-worlds-third-biggest-tech-startup-hub-study-298

QUESTION 50.
Consider following statements with reference to 'Convention on International Trade in Endangered Species
of Wild Fauna and Flora'(CITES)

1. CITES is an international agreement between governments

2. Its aim is to ensure international trade in specimens of wild animals and plants does not threaten their
survival

3. It is not legally binding on the Parties and it does not take the place of national laws.

Which of the statements given above is/are correct?

a) 1 and 2 only
b) 2 only
c) 1 and 3 only
d) 1, 2 and 3 only
Correct Answer: A
Your Answer: Unanswered
Explanation

Ans. (a)

Civilsdaily
Page 0
Email: hello@civilsdaily.com
Exam Title : TS7

Email : pandurangchormale@gmail.com

Contact : 9871159761

Explanation: CITES is an international agreement between governments, to ensure international trade in


specimens of wild animals and plants does not threaten their survival.

The trade in wild animals and plants crosses borders between countries so the effort to regulate it requires
international cooperation.

CITES is legally binding on the Parties (they have to implement the Convention), but it does not take the
place of national laws.

Source: Aug 23

http://www.livemint.com/Politics/EqYlREVQkGzNeb4Z1pLKBN/Proposed-wildlife-law-plans-penalties-of-up-to-Rs50-lakh

QUESTION 51.
Consider the following statements with regard to 'Consumer Protection Councils'.

1. It was established under the Consumer Protection Act, 1986

2. Its aim is to spread consumer awareness and to promote and protect the rights of the consumers

3. It is chaired by Minister of Finance.

Which of the statements given above is/are correct?

a) 1 and 2 only
b) 2 only
c) 1 and 3 only
d) 1, 2 and 3 only
Correct Answer: A
Your Answer: Unanswered
Explanation

Ans. (a)

Explanation: Consumer Protection Act, 1986 is an Act of the Parliament to protect the interests of
consumers in India. It makes provision for the establishment of consumer councils and other authorities for
the settlement of consumers' disputes and for matters connected therewith.

Central Consumer Protection Councils is chaired by Union Minister for Consumer Affairs.

Source: www.civilsdaily.com/lets-know-consumer-protection-councils/

Civilsdaily
Page 0
Email: hello@civilsdaily.com
Exam Title : TS7

Email : pandurangchormale@gmail.com

Contact : 9871159761

QUESTION 52.
Consider the following statements with regard to 'Food Safety and Standards Authority of India (FSSAI)'
which is sometimes in the news.

1. It is non-statutory body established under the Food Safety and Standards Act, 2006

2. It lays down science based standards for various food products in order ensure availability of safe and
wholesome food

3. It operates under aegis of Ministry of Food Safety and Health Standards

Which of the statements given above is/are correct?

a) 1 and 2 only
b) 2 only
c) 1 and 3 only
d) 1, 2 and 3 only
Correct Answer: B
Your Answer: Unanswered
Explanation

Ans. (b)

Explanation: FSSAI established under the Food Safety and Standards Act, 2006 and operates under
aegis of Union Ministry of Health & Family Welfare. It lays down science based standards for various food
products in order ensure availability of safe and wholesome food.

Titbits: It is led by a non-executive Chairperson. The executive head of FSSAI is the Chief Executive
Officer. It has 4 referral laboratories and 72 local laboratories located across India.

Tikdams: There is no such type of Ministry (Ministry of Food Safety and Health Standards).

Source: Aug 23

http://www.livemint.com/Politics/8621Grfw5wxzrnPDjPgyqM/FSSAI-wants-to-regulate-quality-of-tap-water.html

QUESTION 53.
Consider following statements with reference to 'Hague Abduction Convention'

1. It is a multilateral treaty provides an expeditious method to return a child internationally abducted by a


parent from one member country to another

Civilsdaily
Page 0
Email: hello@civilsdaily.com
Exam Title : TS7

Email : pandurangchormale@gmail.com

Contact : 9871159761

2. India recently adopted this convention as an effective way to deal with abduction cases.

Which of the statements given above is/are correct?

a) 1 only
b) 2 only
c) Both 1 and 2
d) Neither 1 and 2
Correct Answer: A
Your Answer: Unanswered
Explanation

Ans. (a)

Explanation: The Hague Convention on the Civil Aspects of International Child Abduction or Hague
Abduction Convention is a multilateral treaty developed by the Hague Conference on Private International
Law (HCCH) that provides an expeditious method to return a child internationally abducted by a parent
from one member country to another.

The Convention was concluded 25 October 1980 and entered into force between the signatories on 1
December 1983.

The Convention was drafted to ensure the prompt return of children who have been abducted from their
country of habitual residence or wrongfully retained in a contracting state not their country of habitual
residence.

Titbit: India has not adopted this convention, so it is still not party to Hague convention.

Recently, US has urged India to join the Hague Abduction Convention to create a more effective response
to deal with such cases. Almost a hundred children born to Indian-American couples are facing an
uncertain future due to the trauma of separation of their parents and the complex legal issues involved

Source:

www.civilsdaily.com/join-agreement-parental-abduction-u-s/

http://www.thehindu.com/news/national/join-agreement-against-parental-abduction-us/article9023062.ece

QUESTION 54.
The idea of forming the proposed National Water Commission (NWC) has recommended by

a) Mihir Shah committee


b) Lodha Committee
c) T.S.R. Subramanian committee
d) Rangarajan committee

Civilsdaily
Page 0
Email: hello@civilsdaily.com
Exam Title : TS7

Email : pandurangchormale@gmail.com

Contact : 9871159761

Correct Answer: A
Your Answer: Unanswered
Explanation

Ans. (a)

Explanation: The idea of forming the proposed National Water Commission (NWC) by merging the Central
Water Commission (CWC) and the Central Ground Water Board (CGWB), was the key recommendation of
a report submitted by Mihir Shah committee that was tasked with reorganising river water management in
the country.

Titbits:

The Shah committee was set up last year to recommend ways to restructure the CWC, which develops
surface water projects, and the Central Ground Water Board (CGWB), which monitors ground water use
and contamination.

At present, both functions are independent of each other. The idea was to initiate reforms in the CWC and
CGWB for integrated water management, development, planning, water-use efficiency and for budgeting
the adoption of a river basin approach. The Shah committee report is the third major report that has been
issued on the restructuring of CWC since 2000.

Source: Aug 24

www.civilsdaily.com/govt-panel-moots-national-water-commission/

http://www.thehindu.com/news/national/pmo-open-to-national-water-commission/article9022684.ece

QUESTION 55.
Recently, Unified Payments Interface (UPI) was in the news. What is it?

1. It is a unique identifier that banks use to transfer money and make payments using the IMPS (Immediate
Payments Service).

2. UPI was launched by National Payments Corporation of India with support from Reserve Bank of India
(RBI)

3. UPI will allow a customer to have multiple virtual addresses for multiple accounts in various banks

Which of the statements given above is/are correct?

a) 1 and 2 only
b) 2 only
c) 1 and 3 only
d) 1, 2 and 3 only

Civilsdaily
Page 0
Email: hello@civilsdaily.com
Exam Title : TS7

Email : pandurangchormale@gmail.com

Contact : 9871159761

Correct Answer: D
Your Answer: Unanswered
Explanation

Ans. (d)

Explanation: Unified Payment Interface (UPI): An improved version of IMPS with only requirements being
a bank account and a smartphone. Once a person registers for UPI with their bank, a unique virtual
address will be created, which is mapped with their mobile phone

Titbits: To initiate the payment, UPI invokes this virtual identity of the beneficiary and transfers money in
real-time. UPI will allow a customer to have multiple virtual addresses for multiple accounts in various
banks. In order to ensure privacy of customers data, there is no account number mapper anywhere other
than the customers own bank.

Source: Aug 25

www.civilsdaily.com/unified-payments-interface-goes-live/

http://www.thehindu.com/business/Industry/unified-payments-interface-goes-live/article9032519.ece

QUESTION 56.
Consider the following statements with regard to 'Masala bonds'.

1. They are financial instruments through which Indian entities can raise money from overseas markets in
foreign currency.

2. First overseas rupee bonds were issued in 2013 by the International Finance Corporation of World Bank.

Which of the statements given above is/are correct?

a) 1 only
b) 2 only
c) Both 1 and 2
d) Neither 1 nor 2
Correct Answer: B
Your Answer: Unanswered
Explanation

Ans. (b)

Civilsdaily
Page 0
Email: hello@civilsdaily.com
Exam Title : TS7

Email : pandurangchormale@gmail.com

Contact : 9871159761

Explanation: Masala bonds are rupee-denominated borrowings by Indian entities in overseas markets.
Usually, while borrowing in overseas markets, the currency is a globally accepted one like dollar, euro or
yen.

Titbits: Advantage of borrowing abroad in rupees - Companies issuing masala bonds do not have to worry
about rupee depreciation, which is usually a big worry while raising money in overseas markets. If the
rupee weakens by the time the bonds come up for redemption, the borrower (company) will need to shell
out more rupees to repay the dollars.

Source:

www.civilsdaily.com/banks-issue-masala-bonds-rbi-opens-currency-markets/

http://www.moneycontrol.com/news/bonds-news/explained-whatmasala-bondshow-do-indian-cos-benefit_7043101.html

QUESTION 57.
Recently, the Cabinet approved the 'Surrogacy (Regulation) Bill 2016', consider the following statements
with regard to bill.

1. A woman will be allowed to become a surrogate mother only for altruistic purpose and under no
circumstances money shall be paid to her, except for medical expenses.

2. The legally-wedded Indian couples (for minimum 5 years) and overseas Indians can have children
through surrogacy.

3. Foreigners, homosexual couples, people in live-in relationships and single individuals are barred from
commissioning surrogacy.

Which of the statements given above is/are correct?

a) 1 and 2 only
b) 3 only
c) 1 and 3 only
d) 1, 2 and 3 only
Correct Answer: C
Your Answer: Unanswered
Explanation

Ans. (c)

Civilsdaily
Page 0
Email: hello@civilsdaily.com
Exam Title : TS7

Email : pandurangchormale@gmail.com

Contact : 9871159761

Explanation: The draft surrogacy Bill aims at regulating commissioning of surrogacy in the country in a
proper manner. The Bill aims to prevent exploitation of women, especially those in rural and tribal areas.
The Bill promises to ensure parentage of children born out of surrogacy is legal and transparent.

Overseas Indians, single parents, live-in partners and homosexuals, are barred from commissioning
surrogacy.

Only legally-wedded Indian couples can have children through surrogacy, provided at least one of them
have been proven to have fertility-related issues.

Source: Aug 24

http://www.thehindu.com/news/national/cabinet-clears-surrogacy-bill/article9025848.ece

QUESTION 58.
Which of the following statements correctly describes "Bharatiya Muslim Mahila Andolan" (BMMA)

a) Private Muslim organization which fights for justice and equality in women entrepreneurship
b) Rights-based mass organization led by Muslim women, fights for citizenship rights of Muslims in India
c) Secular non-government organisation fights for all women in India
d) Rights-based Women government organisation fights for rights of muslim women
Correct Answer: B
Your Answer: Unanswered
Explanation

Ans. (b)

Explanation: Bharatiya Muslim Mahila Andolan(BMMA) is an autonomous, secular, rights-based mass


organization led by Muslim women which fights for the citizenship rights of the Muslims in India. The BMMA
was formed in January 2007. The organisation is based in Mumbai.

Titbits: BMMA conducted a Study of Muslim womens views on reforms in Muslim personal law 'Seeking
Justice Within the Family' across 10 states that revealed that an overwhelming 82% of the over 4,000
women who were surveyed had no property in their name and that 78% were home makers with no income
of their own. The BMMA has demanded a ban on the practice of 'Triple Talaq' (verbal divorce).

Tikdams: Option 'd' is a red flag as it is an andolan/movement, so there is no role of government. Think of
option 'b' as recently, triple talaq, property rights issues, temple entry and personal law reforms was
frequently in the news.

Recently, Bharatiya Muslim Mahila Andolan had filed a PIL against the ruling of Haji Ali dargah trust, and
urged the HC to intervene and overthrow the trusts diktat.(Women temple entry)

Civilsdaily
Page 0
Email: hello@civilsdaily.com
Exam Title : TS7

Email : pandurangchormale@gmail.com

Contact : 9871159761

Source: Aug 26

http://www.livemint.com/Politics/AgErifIXolwSm9vS18CV1L/Haji-Ali-row-Bombay-HC-allows-womens-entry-in-the-darga

QUESTION 59.
Recently, ISRO successfully test-fired a Scramjet (or Supersonic Combusting ramjet) engine. Consider
following statements with regard to Scramjet engine.

1. A scramjet is a variant of a ramjet airbreathing jet engine in which combustion takes place in supersonic
airflow.

2. It contributes to smaller launch vehicles with more payload capacity and promises cheaper access to
outer space.

3. India became the second country to demonstrate the flight testing of a scramjet engines.

Which of the statements given above is/are correct?

a) 1 and 2 only
b) 2 only
c) 1 and 3 only
d) 1, 2 and 3 only
Correct Answer: A
Your Answer: Unanswered
Explanation

Ans. (a)

Explanation: A scramjet (supersonic combusting ramjet) is a variant of a ramjet airbreathing jet engine in
which combustion takes place in supersonic airflow. India became the fourth country to demonstrate the
flight testing of a scramjet engines. This mission is a milestone for ISROs future space transportation
system.

Titbits: Conventional rocket engines need to carry both fuel and oxidiser on board for combustion to
produce thrust, scramjets obtain oxygen from the atmosphere by compressing the incoming air before
combustion at hypersonic speed. It can also liquefy the oxygen and store it on board. Also, the post-flight
analysis of the RLV-TD test flight (utilising scramjet technology) had shown encouraging results.

Source: Aug 29

Civilsdaily
Page 0
Email: hello@civilsdaily.com
Exam Title : TS7

Email : pandurangchormale@gmail.com

Contact : 9871159761

http://indianexpress.com/article/technology/science/isros-scramjet-technology-why-is-it-important-for-the-space-agency-

QUESTION 60.
Consider the following statements with regard to Doha development agenda at WTO.

1. It is the WTO round of trade negotiations among the members started at Doha in 2001.

2. It aims to sign a pact to open up world trade by lowering or eliminating trade barriers.

3. It will help in improving the trade prospects of developing and poor countries, and remove the distortions
in world trade.

Which of the statements given above is/are correct?

a) 1 and 2 only
b) 2 only
c) 1 and 3 only
d) 1, 2 and 3 only
Correct Answer: D
Your Answer: Unanswered
Explanation

Ans. (d)

Explanation: The latest round of trade negotiations among the WTO members started at Doha in 2001.
The negotiation at this round is known as the Doha Development Agenda.

Recently, India has taken lead in WTO-level talks to open up global services trade in Oslo meet. Especially
to ensure easier movement of skilled professionals for short-term projects overseas.

Titbits: The Oslo meeting of finance ministers is likely to discuss the so-called new issues considered
important by the rich nations in todays global trade.

The New Issues include: Clean energy efforts to reach an agreement at the WTO-level to eliminate tariffs
on environmental & sustainable goods produced using clean & green energy and, Global Value Chains &
promotion of supply chains, impact of Brexit on global trade.

Source: Aug 28

http://www.thehindu.com/business/Economy/amid-growing-protectionism-global-trade-ministers-to-meet/article9043058

Civilsdaily
Page 0
Email: hello@civilsdaily.com
Exam Title : TS7

Email : pandurangchormale@gmail.com

Contact : 9871159761

QUESTION 61.
Consider the following statements with reference to The Logistics Exchange Memorandum of Agreement
(LEMOA), which was recently signed by India and U.S.

1. It allows both sides access to each others military facilities for refuelling and replenishment

2. The agreement covers four areas port calls, joint exercises, training and Humanitarian Assistance and
Disaster Relief.

3. It also provides automatic access to the use of military bases to each others military.

Which of the statements given above is/are correct?

a) 1 and 3 only
b) 2 only
c) 1 and 2 only
d) 1, 2 and 3 only
Correct Answer: C
Your Answer: Unanswered
Explanation

Ans. (c)

Explanation: LEMOA stands for Logistics Exchange Memorandum of Agreement (LEMOA), a tweaked
India-specific version of the Logistics Support Agreement (LSA), which the U.S. has with several countries
with whom it has close military to military cooperation.

It does not provide automatic access to the use of military bases to each others military area.

Titbits: India and the U.S. already hold large number of joint exercises during which payments are done
each time, which is a long and tedious process.

Important: This is not a basing agreement. There will be no basing of the U.S. troops or assets on Indian
soil. This is purely a logistical agreement. India can access the string of U.S. facilities across the globe for
logistical support and the U.S., which operates in a big way in Asia-Pacific, will benefit from Indian facilities.
India will also benefit due to its growing global footprint.

Source: Aug 28

www.civilsdaily.com/india-u-s-sign-logistics-agreement/

http://www.thehindu.com/news/national/india-us-set-to-sign-key-logistics-agreement/article9042890.ece

Civilsdaily
Page 0
Email: hello@civilsdaily.com
Exam Title : TS7

Email : pandurangchormale@gmail.com

Contact : 9871159761

QUESTION 62.
India is planning a major diplomatic outreach in order to push through the Comprehensive Convention on
International Terrorism (CCIT) in the United Nations. Consider the following statements in this regard.

1. It is a draft proposed by India in 1996 that is yet to be adopted by the United Nations General Assembly
(UNGA)

2. It intends to criminalize international terrorism and deny terrorists, their financiers and supporters access
to funds, arms, and safe havens.

Which of the statements given above is/are correct?

a) 1 only
b) 2 only
c) Both 1 and 2
d) Neither 1 nor 2
Correct Answer: C
Your Answer: Unanswered
Explanation

Ans. (c)

Explanation: Recently, India is planning a major diplomatic outreach ahead of the 71st session of the
United Nations General Assembly in order to push through the Comprehensive Convention on
International Terrorism (CCIT).

Titbits: It calls for -

-Universal definition of terrorism: no good terrorist or bad terrorist.

-Ban on all groups regardless of country of operation, cut off access to funds and safe havens.

-Prosecution of all groups including cross border groups.

-Amending domestic laws to make cross-border terror an extraditable offence.

-It also addresses, among other things, the issue of Pakistans alleged support for cross-border terrorism in
south Asia.

Source: www.civilsdaily.com/india-plans-anti-terror-drive-nam/

http://www.thehindu.com/news/national/india-plans-antiterror-drive-at-nam/article9043112.ece

Civilsdaily
Page 0
Email: hello@civilsdaily.com
Exam Title : TS7

Email : pandurangchormale@gmail.com

Contact : 9871159761

QUESTION 63.
Consider following statements in regard to 'Foreign Contribution Regulation Act (FCRA)'.

1. It regulates the foreign contribution and foreign hospitality given to NGOs.

2. Finance Ministry is responsible for implementing Foreign Contribution Regulation Act (FCRA)

Which of the statements given above is/are correct?

a) 1 only
b) 2 only
c) Both 1 and 2
d) Neither 1 nor 2
Correct Answer: A
Your Answer: Unanswered
Explanation

Ans. (a)

Explanation: The Foreign Contribution (regulation) Act, 2010 is an Act of Parliament, by the 42nd Act of
2010. Objective is to regulate the acceptance and utilisation of foreign contribution or foreign hospitality by
certain individuals or associations or companies and to prohibit acceptance and utilisation of foreign
contribution or foreign hospitality for any activities detrimental to the national interest and for matters
connected therewith or incidental thereto.

Home Ministry is responsible for implementing Foreign Contribution Regulation Act (FCRA), 2010.

Titbits: Recently, Finance Ministry regulates International donors such as the Ford Foundation which are
registered under FEMA but not the Foreign Contribution Regulation Act (FCRA), 2010.

Home Ministry monitors foreign funds donated to NGOs and organisations through the FCRA.

Source: Aug 29

www.civilsdaily.com/mha-oversight-ngos/

http://www.thehindu.com/news/national/mha-oversight-for-all-ngos/article9043107.ece?homepage=true

QUESTION 64.
Union government has set up a ministerial team to take a final decision on the issue of introducing totaliser
machines. What are totaliser machines?

a) It is new mathematical machine model invented by NASA


b) It mixes votes from various booths for counting votes for elections
c) It is new computing method for counting complex arithmatic operations in calculator
d) New machines for counting statistics of weather forecast by meterological department

Civilsdaily
Page 0
Email: hello@civilsdaily.com
Exam Title : TS7

Email : pandurangchormale@gmail.com

Contact : 9871159761

Correct Answer: B
Your Answer: Unanswered
Explanation

Ans. (b)

Explanation: Totaliser machines mixes votes from various booths for counting. In the current system of
election voting, votes from each Electronic Voting machines (EVM) are counted separately & thus it reveals
the voting trends in each polling station. This leaves the voters in that vicinity open to harassment,
intimidation and post-election victimisation

Advantages are preventing disclosure of voting patterns across polling stations. This will allay the fears of
voters against any pre-poll intimidation or post-poll victimisation by any candidate.

Election Commission had proposed totaliser machines in November 2008 & the proposal was seconded by
the Law Commission in 2015. So, it was frequently in news.

Source: Aug 30

www.civilsdaily.com/centre-moves-vote-totaliser-machines/

http://www.thehindu.com/news/national/centre-moves-on-vote-totaliser-machines/article9046763.ece

QUESTION 65.
Consider following statement with regard to Deendayal Antyodaya Mission- National Rural Livelihoods
Mission (DAY-NRLM).

1. NRLM was launched in 2011 to expand livelihoods in urban areas in agriculture, small enterprises and
formal sector employment.

2. Its objective is to organise all rural poor households and continuously nurture and support them till they
come out of abject poverty.

3. It also provides umbrella cover to Mahila Kisan Sashaktikaran Pariyojana to meet specific needs of
women farmers.

Which of the statements given above is/are correct?

a) 1 and 3 only
b) 1 and 2 only
c) 2 and 3 only
d) 1, 2 and 3 only

Civilsdaily
Page 0
Email: hello@civilsdaily.com
Exam Title : TS7

Email : pandurangchormale@gmail.com

Contact : 9871159761

Correct Answer: C
Your Answer: Unanswered
Explanation

Ans. (c)

Explanation: NRLM was launched in 2011 to expand livelihoods in rural areas in agriculture, small
enterprises and formal sector employment.

DAY-NRLM has the objective to organise all rural poor households and continuously nurture and support
them till they come out of abject poverty.

Titbits: It works for bridging gaps between industries and skilled labour, a placement cell linked to Deen
Dayal Upadhyay Grameen Kaushalya Yojana is also implemented. It provides umbrella cover to Mahila
Kisan Sashaktikaran Pariyojana to meet specific needs of women farmers and achieve socio-economic and
technical empowerment of rural women farmers.

Tikdams: Option 1 - red flagged as it talks about scheme for urban areas instead of rural. Simply,
eliminating option 1 we get answer.

Source: Yojana - August 2016

QUESTION 66.
Which of the following correctly describes objective of 'USTTAD scheme'?

a) To provide training for developing skills of tribals from north-eastern region


b) Upgrading skills and training of minority communities by preservation of traditional ancestral arts and
crafts.
c) To empower women economically from south-western region by providing them skill training
d) To promote bank financing for start-up ventures to boost women entrepreneurship
Correct Answer: B
Your Answer: Unanswered
Explanation

Ans. (b)

Explanation: USTTAD scheme is Upgrading Skills and Training in Traditional Arts/Crafts for Development.
It aims at upgrading skills and training of minority communities by preservation of traditional ancestral arts
and crafts. It envisages boosting the skills of craftsmen, weavers and artisans who are already engaged in
work.
Scheme is implemented by Ministry of Minority Affairs.

Civilsdaily
Page 0
Email: hello@civilsdaily.com
Exam Title : TS7

Email : pandurangchormale@gmail.com

Contact : 9871159761

Source: Yojana - August 2016

QUESTION 67.
Consider following statements with regard to National Apprenticeship Promotion scheme.

1. It provides industry led, practice oriented, effective and efficient mode of formal

training.

2. It has target of 50 lakh apprentices trained by 2019-20.

3. It is implemented under aegis of Ministry of Finance.

Which of the statements given above is/are correct?

a) 1 and 2 only
b) 3 only
c) 2 and 3 only
d) 1, 2 and 3 only
Correct Answer: A
Your Answer: Unanswered
Explanation

Ans. (a)

Explanation: National Apprenticeship Promotion scheme is implemented by Ministry of Skill


development and Entrepreneurship. 25% of total stipend payable will be shared with employers by GoI.
It also supports basic training which is essential component of apprenticeship training. Expected to become
one of the most powerful skill-delivery vehicles in country.

Titbits: Apprenticeship training is considered to be the most efficient ways to develop skilled manpower of
country.

-It provides industry led, practice oriented, effective and efficient mode of formal training.

-National Policy for Skill development has one of its focus as apprenticeship training

-The policy proposes to work pro-actively with the industry including MSMEs to facilitate tenfold increase
opportunities by 2020

Source: Yojana - August 2016

QUESTION 68.

Civilsdaily
Page 0
Email: hello@civilsdaily.com
Exam Title : TS7

Email : pandurangchormale@gmail.com

Contact : 9871159761

Consider following statements about 'Suryamitra' app.

1. It is developed by Infosys with support from Ministry of New & Renewable Energy

2. This App is a high end technology platform which can handle thousands of calls simultaneously and can
efficiently monitor all visits of Suryamitras.

3. It helps to enhance the employment of trained youth i.e. Suryamitras in solar PV technology.

Which of the statements given above is/are correct?

a) 1 and 2 only
b) 3 only
c) 2 and 3 only
d) 1, 2 and 3 only
Correct Answer: C
Your Answer: Unanswered
Explanation

Ans. (c)

Explanation: Suryamitra mobile app is developed by National Institute of Solar Energy (NISE), an
autonomous institution of Ministry of New & Renewable Energy (MNRE). Using this app, customers can
avail services like maintenance, serving and repairing at the click of a button on their mobiles.

It will help enhance the employment of trained youth i.e. Suryamitras in solar PV technology. Further it will
also help to improve the businesses of solar entrepreneurs.

Source: Yojana - August 2016

QUESTION 69.
Which of the following sources of gas have been categorized as un-conventional gas sources?

1) Coal bed methane (CBM)

2) Coal mine methane (CMM)

3) Shale gas

4) Tight gas

Select the correct answer using the codes given below.


a) 1 and 2
b) 2, 3 and 4 only
c) 1 and 3
d) All of the above

Civilsdaily
Page 0
Email: hello@civilsdaily.com
Exam Title : TS7

Email : pandurangchormale@gmail.com

Contact : 9871159761

Correct Answer: D
Your Answer: Unanswered
Explanation

Ans (d)

Explanation: Traditionally, the Coal bed methane (CBM), Coal mine methane (CMM), Shale gas, Tight gas
have been categorized as un-conventional gas sources.

Titbits: The global gas demand is likely to go up by 50% in 2035 and about 1/3 of this will come through
unconventional sources. Shale oil and gas production has been rising across the globe, especially in the
US, which has led to the U.S becoming a net exporter of gas.

3 factors have come together in recent years to make shale gas production economically viable and was
frequently in the news:

Technological advance in horizontal drilling, Hydraulic fracturing, Increase in natural gas price in the global
market.

Source: Yojana - August 2016

QUESTION 70.
Consider the following statements with regard to the Competition Commission of India (CCI).

1. It is a constitutional autonomous body established under The Competition Act of 2002.

2. Objective is to eliminate practices that adversely affect competition in different industries and protect
interests of consumers and ensure freedom of trade.

Which of the statements given above is/are correct?

a) 1 only
b) 2 only
c) Both 1 and 2
d) Neither 1 and 2
Correct Answer: B
Your Answer: Unanswered
Explanation

Ans. (b)

Civilsdaily
Page 0
Email: hello@civilsdaily.com
Exam Title : TS7

Email : pandurangchormale@gmail.com

Contact : 9871159761

Explanation: It is statutory body established under The Competition Act of 2002. CCI consists of a
Chairperson and 6 Members appointed by the Central Government. It is the duty of the Commission to
eliminate practices having adverse effect on competition, promote and sustain competition, protect the
interests of consumers and ensure freedom of trade in the markets of India.

The Commission is also required to give opinion on competition issues on a reference received from a
statutory authority established under any law and to undertake competition advocacy, create public
awareness and impart training on competition issues.

Tikdams: Option 1 straightforwardly eliminated as any executive or advisory body established under an
Act is known as statutory body. In order to have constitutional body, there is need of constitutional
amendment by Parliament.

Source: Sept 1

www.civilsdaily.com/cci-fines-cement-firms-cartelisation/

http://www.thehindu.com/business/Industry/cci-fines-cement-firms-for-cartelisation/article9056379.ece

QUESTION 71.
Recently, The Cabinet gave its approval to create a Project Development Fund (PDF). Consider following
statements in this regard.

1. It will be used for catalysing Indian economic presence in Cambodia, Laos, Myanmar and Vietnam.

2. It is to be housed in the Department of External affairs and operated through the EXIM Bank.

3. It will be governed by an inter-ministerial committee under the chairpersonship of the Commerce


Secretary.

Which of the statements given above is/are correct?

a) 1 and 2 only
b) 3 only
c) 1 and 3 only
d) 1, 2 and 3 only
Correct Answer: C
Your Answer: Unanswered
Explanation

Ans. (c)

Civilsdaily
Page 0
Email: hello@civilsdaily.com
Exam Title : TS7

Email : pandurangchormale@gmail.com

Contact : 9871159761

Explanation: The Cabinet gave its approval to create a Project Development Fund (PDF) with a corpus of
Rs.500 Crore for catalysing Indian economic presence in Cambodia, Laos, Myanmar and Vietnam. The
PDF is to be housed in the Department of Commerce and operated through the EXIM Bank.

Source: Sept 1

www.civilsdaily.com/project-development-fund-approved-cabinet/

http://www.thehindu.com/business/Economy/cabinet-nod-for-permanent-residency-to-fdi-investors/article9056380.ece

QUESTION 72.
Consider the following statements with reference to Asian Infrastructure Investment Bank (AIIB).

1. AIIB is an international financial institution head quartered in Shanghai

2. Its aim is to support the building of infrastructure projects in the Africa-Pacific region

Which of the statements given above is/are correct?

a) 1 only
b) 2 only
c) Both 1 and 2
d) Neither 1 nor 2
Correct Answer: D
Your Answer: Unanswered
Explanation

Ans. (d)

Explanation: Asian Infrastructure Investment Bank (AIIB) is an international financial institution head
quartered in Beijing, China. Its aim is to support the building of infrastructure projects in the Asia-Pacific
region.

Titbits: The projects are either co-financed with other developmental institutions or solely funded by AIIB. It
has 57 founding members including India.

Its recent projects are, slum renovation in Indonesia, highway construction in Pakistan and Tajikistan,
power grid upgrade in Bangladesh.

Source: Sept 1

Civilsdaily
Page 0
Email: hello@civilsdaily.com
Exam Title : TS7

Email : pandurangchormale@gmail.com

Contact : 9871159761

www.civilsdaily.com/canada-apply-join-china-backed-aiib-president-aiib/

http://www.thehindu.com/news/international/canada-to-apply-to-join-chinabacked-aiib-president-of-aiib/article9058685.e

QUESTION 73.
Which of the following correctly describes about 'Scorpene-class submarines'?

a) Class of diesel-electric attack submarines.


b) Class of aircraft carrier protection submarines
c) Class of ballistic missile submarines
d) None of the above
Correct Answer: A
Your Answer: Unanswered
Explanation

Ans. (a)

Explanation: The Scorpene-class submarines are a class of diesel-electric attack submarines. These
submarines, which are designed by a French Company DCNS, are currently being built at a
government-run shipyard in Mumbai.

Titbits: Currently, India has 14 submarines that are all nuclear-powered. These are Chakra (Akula II) class,
Sindhughosh class, and Shishumar class. Two more submarines are now under construction, these are of
the Arihant and Kalvari class.

Source: Sept 3

www.civilsdaily.com/what-are-scorpene-submarines/

http://indianexpress.com/article/india/india-news-india/india-shelves-plan-to-expand-french-submarine-order-after-data-b

QUESTION 74.
Consider the following statements about 25th IUCN World Conservation Congress.

1. The IUCN World Conservation congress took place in Gland, Switzerland

2. Theme of IUCN Congress was 'Planet at the crossroads'

Which of the statements given above is/are correct?

a) 1 only
b) 2 only
c) Both 1 and 2
d) Neither 1 nor 2

Civilsdaily
Page 0
Email: hello@civilsdaily.com
Exam Title : TS7

Email : pandurangchormale@gmail.com

Contact : 9871159761

Correct Answer: B
Your Answer: Unanswered
Explanation

Ans. (b)

Explanation: The opening ceremony of the 25th World Conservation Congress of the International Union
for Conservation of Nature (IUCN) took place at the Neal Blaisdell Center in Hawaii. Hawaii is the
endangered species capital of the world. Islands are the frontlines of biodiversity loss and most vulnerable
to climate change.

Titbits: The main issues discussed at the Congress are wildlife trafficking, ocean conservation,
nature-based solutions for climate change mitigation and adaptation, and private investment in
conservation.

The Congress aims to improve how we manage our natural environment for human, social and economic
development.

Source: Sept 3

http://www.livemint.com/Politics/GgQngTlTI0emWDmXnXVQiI/10day-IUCN-World-Conservation-Congress-begins.html

QUESTION 75.
Consider the following statements about 'Universal Service Obligation Fund'.

1. Its aim is to provide telecom services to rural and remote areas at affordable and reasonable prices.

2. It was created by Ministry of Finance.

Which of the statements given above is/are correct?

a) 1 only
b) 2 only
c) Both 1 and 2
d) Neither 1 nor 2
Correct Answer: A
Your Answer: Unanswered
Explanation

Ans. (a)

Civilsdaily
Page 0
Email: hello@civilsdaily.com
Exam Title : TS7

Email : pandurangchormale@gmail.com

Contact : 9871159761

Explanation: It is created under Department of Telecommunications, Ministry of Communication and


Information Technology. It has statutory support under Indian Telegraph (amendment) Act, 2003.

Titbits: Finance comes from the Universal Access Levy charged from the telecom operators as a
percentage of various licenses fees being paid by them. It is a non-lapsable fund i.e., unspent amount
under target financial year does not lapse, accrues for next years spending All credits to the fund require
parliamentary approval.

Source: Sept 5

www.civilsdaily.com/mobile-access-scheme-remote-areas-soon/

http://www.thehindu.com/business/Industry/mobile-access-scheme-for-remote-areas-soon/article9072633.ece

QUESTION 76.
Consider the following statements about 'International Civil Aviation Organization (ICAO)'.

1. It is a specialized agency of the United Nations, established by Chicago Convention

2. Its headquarters are located in the Quartier International of Montreal, Canada.

Which of the statements given above is/are correct?

a) 1 only
b) 2 only
c) Both 1 and 2
d) Neither 1 nor 2
Correct Answer: C
Your Answer: Unanswered
Explanation

Ans. (c)

Explanation: The International Civil Aviation Organization (ICAO) is a UN specialized agency, established
to manage the administration and governance of the Convention on International Civil Aviation (Chicago
Convention).

Titbits: The Convention on International Civil Aviation, drafted in 1944 by 54 nations, was established to
promote cooperation and create and preserve friendship and understanding among the nations and
peoples of the world. Known more commonly today as the Chicago Convention, this landmark agreement
established the core principles permitting international transport by air, and led to the creation of the

Civilsdaily
Page 0
Email: hello@civilsdaily.com
Exam Title : TS7

Email : pandurangchormale@gmail.com

Contact : 9871159761

specialized agency which has overseen it ever since the International Civil Aviation Organization (ICAO).

Source: Sept 6

http://www.livemint.com/Politics/DyieeQOXPMZLv9Rjz2AZBJ/China-US-Europe-pledge-support-for-global-aviation-emis

http://www.icao.int/about-icao/History/Pages/default.aspx

QUESTION 77.
Consider the following statements with reference to Preferential trade agreement (PTA).

1. It is a trading bloc that gives preferential access to certain products from the participating countries.

2. PTA is called the first stage of economic integration among countries.

Which of the statements given above is/are correct?

a) 1 only
b) 2 only
c) Both 1 and 2
d) Neither 1 nor 2
Correct Answer: C
Your Answer: Unanswered
Explanation

Ans. (c)

Explanation: A preferential trade area (also preferential trade agreement, PTA) is a trading bloc that gives
preferential access to certain products from the participating countries. This is done by reducing tariffs but
not by abolishing them completely. A PTA can be established through a trade pact. It is the first stage of
economic integration.

The line between a PTA and a free trade area (FTA) may be blurred, as almost any PTA has a main goal of
becoming a FTA in accordance with the General Agreement on Tariffs and Trade (GATT).

Source: Sept 6

www.civilsdaily.com/govt-signs-agreement-widen-trade-pact-chile/

http://www.business-standard.com/article/economy-policy/govt-signs-agreement-to-widen-trade-pact-with-chile-1160906

Civilsdaily
Page 0
Email: hello@civilsdaily.com
Exam Title : TS7

Email : pandurangchormale@gmail.com

Contact : 9871159761

QUESTION 78.
Consider the following statements about G20.

1. G20 was founded in 1999 with the aim of promoting high-level discussion of policy issues pertaining to
the promotion of international financial stability.

2. The G20 operates its permanent secretariat in Paris.

Which of the statements given above is/are correct?

a) 1 only
b) 2 only
c) Both 1 and 2
d) Neither 1 nor 2
Correct Answer: A
Your Answer: Unanswered
Explanation

Ans. (a)

Explanation: The G20 (Group of Twenty) is an international forum for the governments and central bank
governors from 20 major economies. It was founded in 1999 with the aim of studying, reviewing, and
promoting high-level discussion of policy issues pertaining to the promotion of international financial
stability.

The G20 operates without a permanent secretariat or staff. The group's chair rotates annually among the
members and is selected from a different regional grouping of countries.

Source: Sept 5

www.civilsdaily.com/g-20-pledge-roll-back-protectionist-measures-2018-end/

http://www.thehindu.com/business/Economy/g20-pledge-to-roll-back-protectionist-measures-by-2018end/article9075263

QUESTION 79.
Consider following statements with regard to 'Open Sky agreement' under new civil aviation policy.

1. Greece will become the first country with an open sky arrangement under new civil aviation policy.

2. India enters into open sky air service agreements (ASA) with SAARC countries and with countries
beyond 5,000 km radius from Delhi.

Civilsdaily
Page 0
Email: hello@civilsdaily.com
Exam Title : TS7

Email : pandurangchormale@gmail.com

Contact : 9871159761

Which of the statements given above is/are correct?

a) 1 only
b) 2 only
c) Both 1 and 2
d) Neither 1 nor 2
Correct Answer: C
Your Answer: Unanswered
Explanation

Ans. (c)

Explanation: India has signed an MoU and initiated air service agreement with Greece to allow unlimited
number of flights into each others countries. Countries sign ASAs through bilateral negotiations to decide
on the number of flights that airlines can fly into each others countries

Titbits: Under the open sky pact, there is no restriction on flights or seats. At present, India has an open
sky agreement with the US. Also, a near open sky agreement with the U.K. under which there are certain
limitations on the number of flights that can be operated at the Mumbai and Delhi Airports.

Source: Sept 7

www.civilsdaily.com/india-allow-unlimited-flights-greece/

http://www.thehindu.com/business/india-to-allow-unlimited-flights-to-and-from-greece/article9080947.ece

QUESTION 80.
Recently, Cabinet approved establishment of Laser Interferometer Gravitational-wave Observatory (LIGO)
project in

a) Kerala
b) Tamil Nadu
c) Maharashtra
d) Gujarat
Correct Answer: C
Your Answer: Unanswered
Explanation

Ans. (c)

Civilsdaily
Page 0
Email: hello@civilsdaily.com
Exam Title : TS7

Email : pandurangchormale@gmail.com

Contact : 9871159761

Explanation: The Laser Interferometer Gravitational-wave Observatory (LIGO) project that was given the
in-principle approval by the Union Cabinet may come up in Maharashtras Hingoli district. Scientists along
with industry members will be engaged in the construction of the eight km-long beam tube at ultra-high
vacuum on a levelled terrain.

Titbits: The LIGO-India will be piloted and overseen by the Department of Atomic Energy and Department
of Science and Technology.

The LIGO-India shall be coordinated and executed by three leading research institutions Punes
Inter-University Centre for Astronomy and Astrophysics (IUCAA), Indores Raja Ramanna Centre for
Advanced Technology (RRCAT) and Gandhinagars Institute for Plasma Research (IPR).

Source: Sept 8

www.civilsdaily.com/first-ligo-lab-outside-u-s-may-come-up-in-maharashtras-hingoli/

http://www.thehindu.com/sci-tech/science/first-ligo-lab-outside-us-may-come-up-in-maharashtras-hingoli/article9085796

QUESTION 81.
Consider the following statements with regard to 'Medical Council of India (MCI)'.

1. It is a constitutional body for establishing uniform and high standards of medical education in India.

2. NITI Aayog has recommended the replacement of Medical Council of India (MCI) with National Medical
Commission (NMC).

Which of the statements given above is/are correct?

a) 1 only
b) 2 only
c) Both 1 and 2
d) Neither 1 nor 2
Correct Answer: B
Your Answer: Unanswered
Explanation

Ans. (b)

Explanation: The Medical Council of India was first established in 1934 under the Indian Medical Council
Act, 1933. The Council was later reconstituted under the Indian Medical Council Act, 1956 that replaced the
earlier Act.

Civilsdaily
Page 0
Email: hello@civilsdaily.com
Exam Title : TS7

Email : pandurangchormale@gmail.com

Contact : 9871159761

Titbits: The Medical Council of India (MCI) is a statutory body for establishing uniform and high standards
of medical education in India. The Council grants recognition of medical qualifications, gives accreditation
to medical schools, grants registration to medical practitioners, and monitors medical practice in India.

Main functions of the Medical Council of India are -

-Establishment and maintenance of uniform standards for undergraduate medical education.

-Regulation of postgraduate medical education in medical colleges accredited by it.

-Recognition of medical qualifications granted by medical institutions in India.

-Accreditation of medical colleges.

Source: Sept 9

www.civilsdaily.com/states-approve-proposal-to-replace-medical-council-of-india-i/

http://www.thehindu.com/news/national/states-approve-proposal-to-replace-medical-council-of-india/article9087055.ece

QUESTION 82.
Consider the following statements about 'Coalition for Epidemic Preparedness Innovations' (CEPI).

1. CEPI is a public-private coalition that aims to derail epidemics by speeding development of vaccines.

2. Its Headquarters is at the Swedish Institute for Public Health, in Stockholm.

3. India is a key member of the CEPI.

Which of the statements given above is/are correct?

a) 1 and 2 only
b) 3 only
c) 1 and 3 only
d) 1, 2 and 3 only
Correct Answer: C
Your Answer: Unanswered
Explanation

Ans. (c)

Civilsdaily
Page 0
Email: hello@civilsdaily.com
Exam Title : TS7

Email : pandurangchormale@gmail.com

Contact : 9871159761

Explanation: CEPI, Coalition for Epidemic Preparedness Innovations, is a "public-private coalition that
aims to derail epidemics by speeding development of vaccines". The concept is to develop early phases of
vaccines without knowing the details for the form in which the infection will appear, but will still cut down the
time to tailor the eventual vaccine to be effective to the epidemic.

Titbits: Its headquarters is at the Norwegian Institute for Public Health, in Oslo. The CEPI will not focus on
diseases that already have sufficient attention, but will be guided by WHOs R&D blueprint (2016), which
lists eleven illnesses to focus on, including Chikungunya, Middle East Respiratory Syndrome (MERS),
Crimean-Congo Hemorrhagic fever

Source: Sept 11

http://www.thehindu.com/sci-tech/science/k-vijay-raghavan-interview/article9094513.ece

QUESTION 83.
Consider following statements about 'Majuli islands'.

1. It has become the first island district of India.

2. Recently, Majuli entered the Guinness Book of World Records as the worlds largest river island.

Which of the statements given above is/are correct?

a) 1 only
b) 2 only
c) Both 1 and 2
d) Neither 1 and 2
Correct Answer: C
Your Answer: Unanswered
Explanation

Ans. (c)

Explanation: Majuli is the biggest river island in the world in the Brahmaputra River, Assam, and the first
island district of the country. It was formed due to course changes by the river Brahmaputra and its
tributaries.

Titbits: Majuli island is accessible by ferries from the city of Jorhat. The island is about 200 km east from
the state's largest city Guwahati. Majuli covers an area of around 880 sq km but due to frequent flooding
in the Brahmaputra river, it suffers heavy erosion. Majuli is the nerve centre of neo-Vaishnavite culture and
has been trying for the World Heritage Site status.

Civilsdaily
Page 0
Email: hello@civilsdaily.com
Exam Title : TS7

Email : pandurangchormale@gmail.com

Contact : 9871159761

Source:
http://www.thehindu.com/todays-paper/tp-in-school/world-heritage-status-for-majuli-soon/article9082215.ece

QUESTION 84.
Consider the following statements in regard with 'Asia Pacific Trade Agreement (APTA)'.

1. It is the oldest preferential trade pact among Asia-Pacific countries, was signed in 1975.

2. It is an initiative under the United Nations Economic and Social Commission for Asia and the Pacific
(UN-ESCAP).

3. India is not a member and party to APTA agreement.

Which of the statements given above is/are correct?

a) 1 and 2 only
b) 3 only
c) 1 and 3 only
d) 1, 2 and 3 only
Correct Answer: A
Your Answer: Unanswered
Explanation

Ans. (a)

Explanation: The Asia-Pacific Trade Agreement (APTA), previously known as the Bangkok Agreement,
was signed in 1975.

Seven Participating States- Bangladesh, China, India, Lao, Mongolia, South Korea, and Sri Lanka are the
parties to the APTA.

Titbits: APTA is open to all members of the United Nations Economic and Social Commission for Asia and
the Pacific, which serves as the APTA Secretariat.

Recently, Union Cabinet approved a move for exchange of tariff concessions under the Asia Pacific Trade
Agreement (APTA), towards expanding trade ties with a few nations in the region, including China.

India is likely to benefit from offers of China and South Korea for duty concessions in sectors including
textiles, chemicals and iron and steel. Industry sources

Source: Sept 12

Civilsdaily
Page 0
Email: hello@civilsdaily.com
Exam Title : TS7

Email : pandurangchormale@gmail.com

Contact : 9871159761

www.civilsdaily.com/cabinet-nod-for-expanded-trade-with-china-asia-pacific-nations/

http://www.thehindu.com/business/Economy/cabinet-nod-for-greater-trade-with-asia-pacific-nations-including-china/artic

QUESTION 85.
Consider the following statements about Genetic Engineering Appraisal Committee (GEAC).

1. It is established under Ministry of Agriculture and Farmers welfare

2. It is the apex body for approval of activities involving large scale use of hazardous microorganisms and
recombinants

Which of the statements given above is/are correct?

a) 1 only
b) 2 only
c) Both 1 and 2
d) Neither 1 nor 2
Correct Answer: B
Your Answer: Unanswered
Explanation

Ans. (b)

Explanation: GEAC is established under Ministry of Environment, Forests and Climate Change. It is the
apex body for approval of activities involving large scale use of hazardous microorganisms and
recombinants.

It is also responsible for approval of proposals relating to release of genetically engineered organisms and
products including experimental field trials.

Titbits: Recently, the GM mustard in question (DMH-11) has been developed by the Centre for Genetic
Manipulation of Crop Plants of Delhi University.

If it gets the green light from the environment ministrys Genetic Engineering Approval Committee (GEAC),
it will be the first GM food crop to be commercially cultivated in India.

Source: www.civilsdaily.com/what-is-genetic-engineering-appraisal-committee-geac/

QUESTION 86.

Civilsdaily
Page 0
Email: hello@civilsdaily.com
Exam Title : TS7

Email : pandurangchormale@gmail.com

Contact : 9871159761

Recently, the 12th edition of the annual bilateral military training exercise - Yudh Abhyas held in
Uttarakhand. This annual bilateral exercise is between India and

a) China
b) U.S.
c) France
d) Israel
Correct Answer: B
Your Answer: Unanswered
Explanation

Ans. (b)

Explanation: Yudh Abhyas is the 12th edition of the annual bilateral military training exercise between
India-US. It took place at Chaubattia in Uttarakhand, close to the China border. It is one of the longest
running joint military training exercises. Both countries host the exercise alternately.

Titbits: The combined exercise Yudh Abhyas 2016 simulates a scenario where both nations are working
together in a counter-insurgency and counter-terrorism (CICT) environment in mountainous terrain under a
United Nations charter.

Source:

www.civilsdaily.com/after-lemoa-its-war-games-now/

http://www.thehindu.com/news/national/after-lemoa-its-war-games-now/article9101353.ece?homepage=true

QUESTION 87.
Which of the following correctly describes about 'Internet Watch Foundation (IWF)'

a) Non-government organisation works for internet governance


b) Most successful hotline in the world at removing child pornography
c) Humanitarian-aid organisation works from social media
d) None of the above
Correct Answer: B
Your Answer: Unanswered
Explanation

Ans. (b)

Civilsdaily
Page 0
Email: hello@civilsdaily.com
Exam Title : TS7

Email : pandurangchormale@gmail.com

Contact : 9871159761

Explanation: Recently, the countrys first-ever hotline to curb sexual abuse of children through the Internet
and to remove child pornographic content online is set through Internet Watch Foundation. Aarambh
Initiative, a network of organisations and individuals working on child protection in the country, has
collaborated with the U.K.-based Internet Watch Foundation (IWF), which is an industry watchdog and is
the most successful hotline in the world at removing child pornography.

Titbits: The hotline in India will be hosted on aarambhindia.org and will enable users to report child sexual
abuse images and videos in a safe and anonymous environment. While the hotline will initially be in English
and Hindi, it will be available in 22 regional languages.

Source: Sept 14

www.civilsdaily.com/hotline-to-curb-child-pornography/

http://www.thehindu.com/news/national/hotline-to-curb-child-pornography/article9105115.ece

QUESTION 88.
Which of the following correctly describes about 'Operation Vatsalya'.

a) To give home and food support for vulnerable and poor people
b) To give temporary home or shelter support for abandoned old-age people
c) To trace missing or abandoned children
d) To give employment support to minority community
Correct Answer: C
Your Answer: Unanswered
Explanation

Ans. (c)

Explanation: Operation Vatsalya is an intensive programme organised jointly by the police and Social
Justice Departments to trace missing or abandoned children, has been launched in Wayanad district. The
project was launched in the State following the success of Operation Smile, a similar project implemented
in Ghaziabad.

It will visit orphanages, childrens homes and other places in the district where missing children from other
districts find refuge. Once such children are identified, they will be produced before the District Child
Welfare Committee and efforts will be made to find their parents.

Tikdams: Option d can be red flag as 'Vatsalya' means parental affection or loved one.

Source: Sept 15

Civilsdaily
Page 0
Email: hello@civilsdaily.com
Exam Title : TS7

Email : pandurangchormale@gmail.com

Contact : 9871159761

www.civilsdaily.com/operation-vatsalya-trace-missing-children/

http://www.thehindu.com/news/national/kerala/operation-vatsalya-to-trace-missing-children/article7663670.ece

QUESTION 89.
Who defines the Indian Standard Time (IST)?

a) National Standard Time organisation of ISRO


b) National Physical Lab of Council of Scientific and Industrial Research (CSIR)
c) Standard time zone wing of Earth Sciences department
d) National Standard Time Lab of Meteorological department
Correct Answer: B
Your Answer: Unanswered
Explanation

Ans. (b)

Explanation: Recently, The Council of Scientific and Industrial Research (CSIR)s National Physical
Laboratory, the organisation that defines the Indian Standard Time (IST), has formally proposed to the
Central government that all Indian computers be legally required to synchronise their clocks to the IST.

The time displayed on laptops or smartphones being derived from multiple American servers would
be a few seconds off from the actual Indian time. The frequent mismatches in the time stamps make it
harder for Indian cyber security experts to investigate Internet-perpetrated frauds.

Source: Sept 16

www.civilsdaily.com/setting-computers-to-ist-is-just-a-matter-of-time/

http://www.thehindu.com/news/national/setting-computers-to-ist-is-just-a-matter-of-time/article9112703.ece

QUESTION 90.
Consider the following statements in regard to 'The Aadhaar (Targeted Delivery of Financial and Other
Subsidies, Benefits and Services) Bill, 2016'.

1. The Bill intends to provide for targeted delivery of subsidies and services to individuals by Aadhaar
numbers.

2. Aadhaar number can also be a proof of citizenship or domicile.

Which of the statements given above is/are correct?

Civilsdaily
Page 0
Email: hello@civilsdaily.com
Exam Title : TS7

Email : pandurangchormale@gmail.com

Contact : 9871159761

a) 1 only
b) 2 only
c) Both 1 and 2
d) Neither 1 and 2
Correct Answer: A
Your Answer: Unanswered
Explanation

Ans. (a)

Explanation: Any public or private entity can accept the Aadhaar number as a proof of identity of the
Aadhaar number holder, for any purpose. But, Aadhaar number cannot be a proof of citizenship or
domicile. Act provides for the establishment of the Unified Identification Authority of India It also provides
for establishment, operation and maintenance of the Central Identity Data Repository.

Titbits: Use of Aadhaar number - To verify the identity of a person receiving a subsidy or a service, the
government may require them to have an Aadhaar number. If a person does not have an Aadhaar number,
government will require them to apply for it, and in the meanwhile, provide an alternative means of
identification.

Offences and penalties: A person may be punished with imprisonment upto three years and minimum fine
of Rs 10 lakh for unauthorised access to the centralized data-base, including revealing any information
stored in it.

No court shall take cognizance of any offence except on a complaint made by the UID authority or a person
authorised by it.

Source:
http://www.business-standard.com/article/pti-stories/uidai-to-notify-regulations-under-aadhaar-act-2016-116091201340_

www.civilsdaily.com/aadhaar-legislation/

QUESTION 91.
Consider the following statements about 'National Company Law Tribunal'(NCLT).

1. It was constituted by the Ministry of Finance under the Companies Act, 2013.

2. Its objective is to delegate inquiry related to proceedings before it, securing assistance of a magistrate or
collector to take possession of property.

Which of the statements given above is/are correct?

Civilsdaily
Page 0
Email: hello@civilsdaily.com
Exam Title : TS7

Email : pandurangchormale@gmail.com

Contact : 9871159761

a) 1 only
b) 2 only
c) Both 1 and 2
d) Neither 1 nor 2
Correct Answer: B
Your Answer: Unanswered
Explanation

Ans. (b)

Explanation: It was constituted to replace the Company Law Board (CLB), and was notified by the
Corporate Affairs Ministry under the Companies Act, 2013. It has been set up at 10 locations, including the
national capital, Mumbai, Kolkata, Hyderabad and Chennai

NCLT has power to delegate inquiry related to proceedings before it, securing assistance of a magistrate or
collector to take possession of property etc.

Titbits: Link NCLT with Satyam scam, as it was one of the biggest corporate frauds, which involves
financial mis-statements to the tune of about Rs 12,320 crore.

Source: Sept 17

www.civilsdaily.com/lets-know-more-about-national-company-law-tribunal/

http://www.thehindu.com/business/no-repeat-of-2g-satyam-if-laws-are-sincerely-implemented-says-nclt-president/article

QUESTION 92.
Which of the following state does not have 'special category' status.

a) Arunachal Pradesh
b) Tripura
c) Uttarakhand
d) Telangana
Correct Answer: D
Your Answer: Unanswered
Explanation

Ans. (d)

Civilsdaily
Page 0
Email: hello@civilsdaily.com
Exam Title : TS7

Email : pandurangchormale@gmail.com

Contact : 9871159761

Explanation: As of now 11 states have special category status, namely, Arunachal Pradesh, Assam,
Manipur, Meghalaya, Mizoram, Nagaland, Sikkim, Tripura, Jammu & Kashmir, Himachal Pradesh, and
Uttarakhand.

Special category status is a classification given by Centre to assist in development of those states that
face geographical & socio-economic disadvantages like hilly terrains, strategic international borders,
economic & infrastructural backwardness and non-viable state finances.

The classification came into existence in 1969 as per the suggestion given by the Fifth Finance
Commission, set up to devise a formula for sharing the funds of Central govt. among all states.

Source: www.civilsdaily.com/14th-finance-commission-on-special-category-status/

http://www.thehindu.com/news/national/andhra-pradesh/niti-aayog-ruled-out-special-status-to-andhra-pradesh-venkaiah

QUESTION 93.
Which of the following are members of 'MERCOSUR', a sub-regional bloc.

1. Argentina

2. Brazil

3. Peru

4. Colombia

Select the correct answer using the codes given below.

a) 1 and 2
b) 2 only
c) 1, 2 and 3
d) 1, 2, 3 and 4 only
Correct Answer: A
Your Answer: Unanswered
Explanation

Ans. (a)

Explanation: MERCOSUR is sub-regional bloc, and objective is to promote free trade and the fluid
movement of goods, people, and currency in members from South America.

Civilsdaily
Page 0
Email: hello@civilsdaily.com
Exam Title : TS7

Email : pandurangchormale@gmail.com

Contact : 9871159761

It is now a full customs union and a trading bloc having full members of Argentina, Brazil, Paraguay,
Uruguay and Venezuela. and Associate states are Bolivia, Chile, Peru, Colombia, Ecuador and Suriname.

Titbits: Mercosur and the Andean Community of Nations are customs unions that are components of a
continuing process of South American integration connected to the Union of South American Nations.

Source: Sept 24

www.civilsdaily.com/brazil-urges-india-to-broaden-mercosur-presence/

http://www.thehindu.com/news/international/brazil-urges-india-to-broaden-mercosur-presence/article9139905.ece

QUESTION 94.
Recently, Five-hundred metres Aperture Spherical Telescope(FAST), the worlds largest radio telescope
has begun functioning in

a) U.S
b) China
c) Japan
d) France
Correct Answer: B
Your Answer: Unanswered
Explanation

Ans. (b)

Explanation: Recently, Five-hundred metres Aperture Spherical Telescope(FAST), the worlds largest
radio telescope has begun functioning in China. Measuring 500-meters in diameter, the radio telescope is
nestled in a natural basin within a stunning landscape of lush green karst formations in southern Guizhou
Province.

Titbits: It surpasses that of the 300-meter Arecibo Observatory in Puerto Rico, a dish used in research on
stars that led to a Nobel Prize. FAST would search for gravitational waves, detect radio emissions from
stars and galaxies and listen for signs of intelligent extraterrestrial life.
The ultimate goal of FAST is to discover the laws of the development of the universe.

Source: www.civilsdaily.com/worlds-largest-radio-telescope-begins-operations/

http://www.thehindu.com/news/international/worlds-largest-radio-telescope-begins-operations/article9147058.ece

QUESTION 95.

Civilsdaily
Page 0
Email: hello@civilsdaily.com
Exam Title : TS7

Email : pandurangchormale@gmail.com

Contact : 9871159761

Consider the following statements about Swadhar scheme.

1. It was launched by Ministry of Women and Child Development.

2. It aims at rehabilitation of women in difficult circumstances.

Which of the statements given above is/are correct?

a) 1 only
b) 2 only
c) Both 1 and 2
d) Neither 1 nor 2
Correct Answer: C
Your Answer: Unanswered
Explanation

Ans. (c)

Explanation: The Swadhar scheme was launched by the Union Ministry of Women and Child
Development in 2002 for rehabilitation of women in difficult circumstances. The scheme provides shelter,
food, clothing and care to the marginalized women/girls who are in need.

Titbits: The beneficiaries include widows deserted by their families and relatives, women prisoners
released from jail and without family support, women survivors of natural disasters, women victims of
terrorist/extremist violence etc. The implementing agencies are mainly NGOs.

There are 311 Swadhar Homes across the country including Gujarat and Maharashtra, and the total
number of women rehabilitated under the scheme in 2014-15 is 4247.

Source: Yojana - Sept 2016

QUESTION 96.
Consider following statements about 'Sakhi - One stop centre scheme'.

1. Sakhi - One stop centre scheme supports women affected from violence.

2. It offers all types of Medical aid, Police assistance, Legal aid and counselling and shelters.

Which of the statements given above is/are correct?

a) 1 only
b) 2 only
c) Both 1 and 2
d) Neither 1 nor 2

Civilsdaily
Page 0
Email: hello@civilsdaily.com
Exam Title : TS7

Email : pandurangchormale@gmail.com

Contact : 9871159761

Correct Answer: C
Your Answer: Unanswered
Explanation

Ans. (c)

Explanation: The Centres One Stop Centre (OSC) scheme is implemented by Ministry of Women and
Child development under the Nirbhaya Fund. It offers all types of Medical aid, Police assistance, Legal aid
and counselling and shelters. It is an initiative for women safety pertaining to strategic areas of prevention,
protection and rehabilitation.

Titbits: OSC centres is a method to addressing violence against women have been operational in the West
for over three decades and in south Asia for over a decade. Most of these models have been evaluated
and there are critical lessons to be learnt from them

Source: Yojana - Sept 2016

QUESTION 97.
Consider following statements about Sarojini Naidu.

1. She was the first women to become the President of Indian National Congress.

2. She was closely associated with the formation of Women's Indian Association and accompanied the
women's voting rights delegation to London.

Which of the statements given above is/are correct?

a) 1 only
b) 2 only
c) Both 1 and 2
d) Neither 1 and 2
Correct Answer: B
Your Answer: Unanswered
Explanation

Ans. (b)

Explanation: She was the second women to become the President of Indian National Congress. But First
Indian women to do so, and First women President of INC was Annie Beasant.

Civilsdaily
Page 0
Email: hello@civilsdaily.com
Exam Title : TS7

Email : pandurangchormale@gmail.com

Contact : 9871159761

Titbits: She was known as 'Nightingale of India'. Naidu served as the first governor of the United Provinces
of Agra and Oudh from 1947 to 1949, the first woman to become the governor of an Indian state. In 1930
during the salt satyagraha, she was one of the women protesters at the Dharsana salt works, Gujarat.
Hundreds of satyagrahis were beaten by soldiers under British command at Dharasana.

Source: Yojana - Sept 2016

QUESTION 98.
Consider the following statements in regard with women contribution in Indian freedom struggle.

1. Aruna Asaf Ali was the first person to hoist the tricolour at Gowalia Tank Maidan in

2. She was called 'Rani Jhansi of 1942'.

Which of the statements given above is/are correct?

a) 1 only
b) 2 only
c) Both 1 and 2
d) Neither 1 nor 2
Correct Answer: C
Your Answer: Unanswered
Explanation

Ans. (c)

Explanation: She is widely remembered for hoisting the Indian National Congress flag at the Gowalia Tank
maidan in Bombay during the Quit India Movement, 1942. After the independence, she remained active in
politics, becoming Delhi's first mayor in 1958. In the 1960s, she successfully started a media publishing
house. She received India's highest civilian award, the Bharat Ratna, posthumously in 1997.

Source: Yojana - Sept 2016

QUESTION 99.
Identify following Women martyrs of Indian freedom struggle.

1. She is known as 'Lakshmi Bai of Nagaland'

2. She was well versed with tactics of Guerilla War and Arm operations.

Select the correct answer using the codes given below.

a) Haipou Jadonang
b) Rani Gaidinliu
c) Kalpana Dutta
d) Sucheta Kriplani

Civilsdaily
Page 0
Email: hello@civilsdaily.com
Exam Title : TS7

Email : pandurangchormale@gmail.com

Contact : 9871159761

Correct Answer: B
Your Answer: Unanswered
Explanation

Ans. (b)

Explanation: Rani Gaidinliu was a Naga spiritual and political leader who led a revolt against British rule in
India. At the age of 13, she joined the Heraka religious movement of her cousin Haipou Jadonang. The
movement later turned into a political movement seeking to drive out the British from Manipur and the
surrounding Naga areas.

Gaidinliu was arrested in 1932 at the age of 16, and was sentenced to life imprisonment by the British
rulers. Jawaharlal Nehru met her at Shillong Jail in 1937, and promised to pursue her release. Nehru gave
her the title of "Rani" ("Queen"), and she gained local popularity as Rani Gaidinliu.

Source: Yojana - Sept 2016

QUESTION 100.
Consider following statements about 'Digital Gender Atlas' for Advancing Girl's Education in India.

1. It is developed by The Ministry of Human Resource Development with support of UNESCO.

2. It Identifies low performing geographic pockets for girls, particularly from marginalised groups.

Which of the statements given above is/are correct?

a) 1 only
b) 2 only
c) Both 1 and 2
d) Neither 1 nor 2
Correct Answer: B
Your Answer: Unanswered
Explanation

Q.100)

Ans. (b)

Explanation: Digital Gender Atlas' for Advancing Girl's Education is developed by The Ministry of Human
Resource Development with support of UNICEF.

Civilsdaily
Page 0
Email: hello@civilsdaily.com
Exam Title : TS7

Email : pandurangchormale@gmail.com

Contact : 9871159761

It will help identify low performing geographic pockets for girls, particularly from marginalised groups such
as scheduled castes, schedule tribes and Muslim minorities, on specific gender related education
indicators.

Titbits: The Atlas is placed on the MHRD website and available and ready to use by States/Districts/Blocks
education administrators or any other interested group.

The Atlas provides comparative analysis of individual gender related indicators over three years and that
enables a visual assessment of the change and an understanding of whether some intervention introduced
in a geography at a particular point in time has worked or not. It is constructed on an open source platform
with an inbuilt scope of updating data by authorized persons to retain its dynamic character.

Source: Yojana - Sept 2016

Civilsdaily
Page 0
Email: hello@civilsdaily.com

You might also like